149
Clinical Immunology and Allergy  A 19-year-old male patient presen ted with lymphocytic interstitial pneumo nia. Lymph nodes histopathological examination exhibited an effaced structure and a diffuse infiltration of small-sized lymphocytic cells compa tible with a low-grade non-Hodgkin 's lymphoma. The surface phenotype of the sample taken corresponded to sample 4. Choose the correct cell type for sample 4. 1)  A mature, MHC class I restricted T-cell  2)  A mature, MHC class II restricted T-cell  3)  A mature B-cell  4)  An immature T-cell  5)  An immature B-cell  

Clinical Immunology and Allergy

Embed Size (px)

Citation preview

Page 1: Clinical Immunology and Allergy

7/25/2019 Clinical Immunology and Allergy

http://slidepdf.com/reader/full/clinical-immunology-and-allergy 1/156

Clinical Immunology and Allergy

 A 19-year-old male patient presented with lymphocytic interstitial pneumonia. Lymph nodes

histopathological examination exhibited an effaced structure and a diffuse infiltration of small-sized

lymphocytic cells compatible with a low-grade non-Hodgkin's lymphoma. The surface phenotype

of the sample taken corresponded to sample 4.

Choose the correct cell type for sample 4.

1)  A mature, MHC class I restricted T-cell 2)  A mature, MHC class II restricted T-cell 

3)  A mature B-cell 4)  An immature T-cell 5)  An immature B-cell 

Page 2: Clinical Immunology and Allergy

7/25/2019 Clinical Immunology and Allergy

http://slidepdf.com/reader/full/clinical-immunology-and-allergy 2/156

The correct answer is choice D.

Thymocytes fall into 4 classes, CD4-, CD8- (immature T-cells); CD4+, CD8+ (immature T-cells);CD4-, CD8+ (mature cytotoxic T-cells) ; and CD4+, CD8- (mature helper T-cells), which are

sometimes called double negative, double positive, and single positive T cells, respectively. Doublenegative cells can differentiate into the other kinds of cells, but that the single positive cells areterminally differentiated. T-cell receptor (TCR) complex contains an accessory molecule, calledCD3, that is essential of signal transduction. All T cells are positive for CD3. Thus sample 4 is aCD4+, CD8+(double positive) immature T cell. Hence the correct answer is choice D.

T cell receptor protein doesn‟t bind directly to the antigen. It only can bind to pieces of antigenmolecules on the surface of antigen-presenting cells (APCs). MHC proteins serve as carriers ofantigenic material on the surface of APCs. In humans, the MHC gene loci is divided into threeregions: Class I, II, and III. Class I and II genes mostly encode cell surface proteins. Class III genesencode secreted and intracellular proteins. Cytotoxic T CD4-, CD8+ cells mostly recognize bits ofantigen in association with Class I MHC proteins (choice A), while helper T CD4+, CD8- cellsmostly bind to antigen fragments associated with Class II MHC proteins (choice B).

Immature B cells express IgM (choice E) while mature B cells express both IgM and IgD on theirsurface (choice C).

Page 3: Clinical Immunology and Allergy

7/25/2019 Clinical Immunology and Allergy

http://slidepdf.com/reader/full/clinical-immunology-and-allergy 3/156

 

T-cell development in thymus

During initial development of T and B cells in the primary lymphoid organs--such as thymus andbone marrow-- many of the self-reactive and useless clones are actually killed off so that they nolonger exist. This process is called selection. Most of the thymocytes (immature T cells) that enterthe thymus die there and only about 2% survive and leave the thymus as mature T cells. Thatmassive cell death occurs due to selection of the right T cells. Positive selection results in MHCrestriction and negative selection results in self tolerance.

1. Pre-T cells leave the bone marrow and migrate to the thymus.

2. Upon reaching the thymus they begin to undergo rearrangement of their β and then α TCRgenes and to express surface TCR. Cells that fail to do this die.

3. Some T cells will express receptors that cannot bind at all to self-MHC proteins. These cells will

die; i.e., only T cells with receptors that bind self-MHC will survive (positive selection to generatecompetent T cells ).

4. Some surviving T cells will express receptors that bind very strongly to self-MHC proteins

on the surface of APC in the thymus. These T cells will also die (negative selection to eliminate

Page 4: Clinical Immunology and Allergy

7/25/2019 Clinical Immunology and Allergy

http://slidepdf.com/reader/full/clinical-immunology-and-allergy 4/156

autoimmunity).

5. Cells that finally survive become mature T cells and exit the thymus into the peripheralcirculation.

There‟s a progession of developmental stages as T cells mature through the thymus, and theychange their surface molecules as they pass through these stages. The cells entering the thymusexpress TCR proteins and CD3. Most first rearrange their beta TCR chain and generate a cellsurface pseudoTCR containing a beta chain and “pre-alpha” chain plus CD3. These pre orimmature T cells are CD4-, CD8-. Subsequently, they switch on both CD4+, CD8+, and the alphachain of the TCR undergoes rearrangement. These immature double positive cells undergo positiveand negative selection, and those that survive undergo a final selective step whereby they loseeither CD4 or CD8 and become “single-positive” T cells that will be either functional T-helper or T-cytotoxic cells respectively. TCR rearrangement and positive selection occurs in the thymic cortexwhile negative selection occurs in the thymic medulla .

 A 40 year old zoo employee was admitted to the emergency after being bitten by a Gaboon Viper

on his hand. He complained of nausea and had an episode of vomiting as well. He was unable to

move his hands and edema was observed over the bitten area. Prothrombin time was 15.53

(normal range 11.0-13.0). International normal ratio (INR) was measured to be 1.2. He was

administered intravenous antibiotics and antivenin was also given. Local debridement and

Page 5: Clinical Immunology and Allergy

7/25/2019 Clinical Immunology and Allergy

http://slidepdf.com/reader/full/clinical-immunology-and-allergy 5/156

fasciotomy were performed the next day.

 Administration of antivenin is an example of which of the following:

1) IgA mediated mucosal immune response to the antigen 2)  Acquired passive immunity 3) Vaccination against snake venom 4)  Anti allergic measures 5) IgE mediated immunity 

The correct answer is choice B

 Acquired passive immunity refers to the process of obtaining serum from immune individuals,concentrating the immunoglobulin fraction to protect a susceptible person. Passive immunity givesimmediate, but short-lived protection. The method of acquired passive immunization is used tocombat active infections of tetanus, infections hepatitis, rabies, snake bit venom etc. In the case ofsnake bite venom, passive immunity is produced by the antitoxin so the serum is called antivenomserum/antivenin.

The Mucosal immune system (choice A) consists of an integrated network of lymphoid tissues andmucous membrane-associated cells and effector molecules. It provides the host protection fromenvironmental antigens and pathogens. The Mucosal immune system includes antibodies, largely

Page 6: Clinical Immunology and Allergy

7/25/2019 Clinical Immunology and Allergy

http://slidepdf.com/reader/full/clinical-immunology-and-allergy 6/156

of IgA isotype, as well as cytokines, chemokines, and their receptors and immunocytes thataccumulate in, or in transit between, various mucosa-associated lymphoid tissues (MALT).

Vaccination (choice C) builds up immunity against a particular antigen/pathogen. Administration of

antivenin only provides immediate clearance of the venom by providing polyvalent antisera (usuallyequine) that was exposed to small doses of snake venom.

Snake venom contains a mixture of proteins and enzymes such as hyaluronidase,phosphodiesterase, ATPases and phospholipase. It is actively cytotoxic and the symptoms are dueto these toxins and not a allergic reaction (choice D).

IgE antibodies (choice E) bind to allergens to cause release of histamines.

Patients suffering from end stage renal failure who are on chronic intermittent hemodialysis (HD),

show an immunodeficiency of both cell mediated and humoral type. It is manifested in the form of

frequent infections, higher incidences of mortality due to infectious complications and low response

to vaccination.

This immune defect is the result of defective T- cell activation and is NOT the result of which of the

Page 7: Clinical Immunology and Allergy

7/25/2019 Clinical Immunology and Allergy

http://slidepdf.com/reader/full/clinical-immunology-and-allergy 7/156

following?

1)   Absence of co-stimulatory signals 2)   Aberrant secretion of cytokines IL-2 and TNF-α 3)  Injury to lymphoid tissue due to uremia 4)  Defect in Antigen presenting cells (APCs)

5)  Complement activation 

The correct answer is choice C.

In patients undergoing HD, both reduced co-stimulation and aberrant expression of cytokines has

been observed. This is mainly induced by uremia and further exacerbated by regular contact ofblood with artificial dialyzer membranes, complement activation and recurrent exposure topyrogens. The HD patients are thought to be in a chronic state of inflammation. The cytokinedifferentiation profile of circulating T cells from HD patients has been found to be dysregulated andis characterized by an increase in TH1 cells, but a normal amount of TH2 cells. Moreover, a higherpercentage of monocytes capable of secreting the TH1 promoting cytokine interleukin 12 (IL-12)compared to normal populations are observed. There is no documented research correlatinglymphoid tissue damage to uremia.

 A defect in the Antigen presenting cells (APCs) results in their exhibiting a decreased co-stimulatoryactivity. This defect can be abrogated in vitro by the application of a co-stimulatory signal of theB7/CD28 pathway (choice A).

Page 8: Clinical Immunology and Allergy

7/25/2019 Clinical Immunology and Allergy

http://slidepdf.com/reader/full/clinical-immunology-and-allergy 8/156

 

In patients on hemodialysis, pro-inflammatory cytokines such as IL-6, TNF-α and IL-1ß areincreased due to the state of chronic inflammation (choice B). Thus, the defect in the T-cellactivation mainly rests in the responses of the Antigen presenting cells (choice D).

 As mentioned earlier, regular contact of blood with artificial dialyzer membranes causes excessivecomplement activation (choice E) which results in a state of chronic inflammation, ultimatelyresulting in immune dysfunction.

Viruses employ various mechanisms to evade the host immune response. The Herpes simplex

virus encodes a protein called ICP47 (an immediate-early protein) that binds to the cytosolic face of

transporter for antigen processing (TAP) and blocks accessibility of TAP to peptides.

Thus, herpes virus infected cells that express ICP47 would undergo which of the following actions:

1) Resist killing by CD8+ virus-specific cytotoxic T lymphocytes 2) Be unable to stimulate virus-specific antibody production by B lymphocytes 3) Not stimulate CD4+ response 4) Resist complement mediated lysis of the immune complex 5)  Avoid detection by macrophages 

Page 9: Clinical Immunology and Allergy

7/25/2019 Clinical Immunology and Allergy

http://slidepdf.com/reader/full/clinical-immunology-and-allergy 9/156

 The correct answer is choice A.

The herpes simplex viruses (HSV) avoid host responses by inhibition of antigen presentation by

infected host cells. HSV-1 and HSV-2 both express a protein that is synthesized shortly after viralreplication called ICP47, which very effectively inhibits the human transporter molecule needed forantigen processing. Inhibition of TAP blocks antigen delivery to class I MHC receptors on HSV-infected cells. This prevents the presentation of viral antigen to CD8+ T cells resulting in thetrapping of empty class I MHC molecules in the endoplasmic reticulum and effectively shuttingdown the CD8+ T-cell response to HSV-infected cells.

When the above mentioned response is inhibited, the antibodies would not recognize the herpesantigen and further antibody production (Choice B).

The herpes virus evades the CTL response, via class I MHC, and not through the CD4+ /class II

MHC response (choice C).

 Antibody-mediated destruction of viruses requires complement activation, that results either indirect lysis of the viral particle or opsonization and elimination of the virus by phagocytic cells. In

Page 10: Clinical Immunology and Allergy

7/25/2019 Clinical Immunology and Allergy

http://slidepdf.com/reader/full/clinical-immunology-and-allergy 10/156

this case, the complement system cannot detect any immune complex, which is not formed (choiceD).

Since herpes virus is an intracellular pathogen, the mechanism of immune response against it does

not involve the macrophages (choice E) that detect extracellular pathogens and engulf them.  A 3 month old baby girl was admitted for work-up for loose stools and rectal bleeding. Small

patches of eczema were seen on her body. Some form of food allergy was suspected by the

admitting physician. The baby had been weaned from breast milk a few weeks ago and was on

formula milk and semisolid food containing dairy products. She was treated conservatively and

improved soon after her admission. The parents had a family history of allergies and asthma. They

were suitably advised on prevention of food allergies in babies and discharged.

Which of the following is a current recommendation of the American Academy of Pediatrics (AAP)

for decreasing the incidence of food allergy in at-risk children?

1) Breast feed exclusively for 12 months 2) If unable to breast feed, use soy-based formulas rather than cow‟s milk formula 3) Include peanut in the lactation diet to induce oral tolerance 4) Delay introduction of solid foods 5) Begin feeding whole cow‟s milk at age 4 months, before allergies develop 

The correct answer is choice D.

Page 11: Clinical Immunology and Allergy

7/25/2019 Clinical Immunology and Allergy

http://slidepdf.com/reader/full/clinical-immunology-and-allergy 11/156

 

The American Academy of Pediatrics (AAP) has put forth some guidelines that, over largepopulation groups, are believed to help prevent food allergies in high-risk children (those withparents or siblings with food allergies or other allergic disorders, like eczema). According to the

guidelines, it is advisable to wait until infants are four to six months old to introduce solid foods.

Breastfeeding exclusively for at least four to six months (not 12 months), if possible, also reducesthe risk of food allergies (choice A).

If breastfeeding isn't possible or workable in some circumstances, hydrolysate formulas - formulasthat have been broken down into simpler proteins that are considered easier to digest areadvisable. At this time, extensively hydrolyzed formulas are considered the most effective forpreventing allergic conditions. Soy formulas and goat's milk may not be good alternatives, becausemany infants that are allergic to cow's milk may also be allergic to soy) (choice B).

Breastfeeding mothers, should also avoid cow‟s milk, eggs, fish, peanuts, and tree nuts from theirown diet, consumption of peanuts by the lactating mother is not advised (choice C). Cow‟s milkshould be avoided in infants till the age of one year (choice E).

Page 12: Clinical Immunology and Allergy

7/25/2019 Clinical Immunology and Allergy

http://slidepdf.com/reader/full/clinical-immunology-and-allergy 12/156

 A 29-year-old woman was diagnosed as having type 1 diabetes mellitus and received insulin aspart

and neutral protamine hagedorn (NPH) insulin. On day 22, she had leg edema and right abdominal

pain. The serum hepatobiliary enzyme levels were markedly elevated. Computed tomography

revealed gallbladder edema. There was no skin eruption at the local injection site. Oral intake of

foods and drinks was ceased and she was given total parenteral nutrition and human regular

insulin. Challenge testing demonstrated that protamine was the cause of her allergy. This is anextremely rare case of protamine allergy resulting in gallbladder edema.

What type of hypersensitivity reaction has this patient experienced?

1) Type III 2) Type Iva 

3) Type Ivb 4) Type Ivc 5) Type Ivd 

The correct answer is choice D.

The hallmarks of a type IV reaction are the delay in time required for the reaction to develop. A typeIV hypersensitive reaction involves the cell-mediated branch of the immune system.

Page 13: Clinical Immunology and Allergy

7/25/2019 Clinical Immunology and Allergy

http://slidepdf.com/reader/full/clinical-immunology-and-allergy 13/156

Type IVc: T cells can also act as effector cells themselves: they emigrate to the tissue and can killtissue cells like hepatocytes or keratinocytes in a perforin/granzyme B- and FasL-dependentmanner. Such reactions occur in most drug-induced delayed hypersensitivity reactions, mostlytogether with other type IV reactions (monocyte, eosinophil or PMN recruitment and activation).Cytotoxic T cells thus play a role in maculopapular or bullous skin diseases as well as inneutrophilic inflammations (acute generalized exanthematous pustulosis – AGEP), and in contact

dermatitis. Type IVc reactions appear to be dominant in bullous skin reactions, where activatedCD8+ T cells kill keratinocytes, but may also be the dominant cell type in hepatitis or nephritis. Inthis case, the gall bladder edema along with hepatic dysfunction point to a type IVc DTH.

Type IVa (choice B) reactions correspond to TH1-type immune reactions: TH1-type T cells activatemacrophages by secreting large amounts of interferon (IFN)- γ, drive the production of complement

fixing antibody isotypes involved in type II and III reactions (IgG1, IgG3), and are co-stimulatory forpro-inflammatory responses (TNF, IL-12) and CD8+ T-cell responses. The T cells promote thesereactions by IFN- γ secretion and possibly other cytokines (TNF, IL-18, etc.).

Type IVb (choice C) corresponds to the TH2-type immune response. TH2 T cells secrete the

cytokines IL-4, IL-13 and IL-5, which promote B-cell production of IgE and IgG4, macrophagedeactivation and mast cell and eosinophil responses. The high production of the Th2 cytokine IL-5leads to an eosinophilic inflammation, which is the characteristic inflammatory cell type in manydrug hypersensitivity reactions. In addition, there is a link to type I reactions, as Th2 cells boost IgEproduction by IL-4/IL-13 secretion. Examples include infestations with nematodes, or an allergicinflammation of the bronchi or nasal mucosa (asthma and rhinitis).

Page 14: Clinical Immunology and Allergy

7/25/2019 Clinical Immunology and Allergy

http://slidepdf.com/reader/full/clinical-immunology-and-allergy 14/156

 

Type IVd: T cells can coordinate (sterile) neutrophilic inflammations as well. Typical exampleswould be sterile neutrophilic inflammations of the skin, in particular AGEP. In this disease, CXCL8-and GM-CSF-producing T cells recruit neutrophilic leukocytes via CXCL8 release and prevent their

apoptosis via GM-CSF release. Besides AGEP, such T-cell reactions are also found in Behçet‟sdisease and pustular psoriasis .

 A type III hypersensitive reaction (choice A) is mediated by the formation of immune complexes andthe ensuing activation of complement. Complement split products serve as immune effectormolecules that elicit localized vasodilation and chemotactically attract neutrophils. Deposition of

immune complexes near the site of antigen entry can induce an Arthus reaction, in which lyticenzymes released by the accumulated neutrophils and the complement membrane attack complexand cause localized tissue damage.

Type I hypersensitivity reaction or allergy is immediate, in response to an antigen and is IgE

antibody mediated. Clinical manifestations of type I reactions include potentially life-threateningsystemic anaphylaxis and localized responses such as hay fever and asthma.

Type II or Cytotoxic Hypersensitivity also involves antibody-mediated reactions. However, the

Page 15: Clinical Immunology and Allergy

7/25/2019 Clinical Immunology and Allergy

http://slidepdf.com/reader/full/clinical-immunology-and-allergy 15/156

immunoglobulin class (isotype) is generally IgG. Type II hypersensitivity may also involvecomplement that binds to cell-bound antibody. The difference here is that the antibodies arespecific for (or able to cross-react with) "self" antigens. When these circulating antibodies react witha host cell surface, tissue damage may result for e.g. autoimmune hemolytic anemia (AHA).

Two polio vaccines are used throughout the world to combat poliomyelitis (or polio). The twovaccines have eradicated polio from most countries in the world, and reduced the worldwide

incidence from an estimated 350,000 cases in 1988 to 1,652 cases in 2007.

 All of the following groups are contraindicated to receive the polio vaccine except?

1)  A 45 year old man who has previously had streptomycin allergy 

2)  A 12 month old girl who is experiencing a kidney infection 3)  A 12 month old boy who exhibited allergy symptoms after receiving his polio shot at 4

months 4)  A 7 year old girl who developed allergy to neomycin when she was 65)  A 32 year old woman who is HIV positive but clinically healthy 

The correct answer is choice E.

Current US recommendations are to vaccinate children and specific groups of adults with the

Page 16: Clinical Immunology and Allergy

7/25/2019 Clinical Immunology and Allergy

http://slidepdf.com/reader/full/clinical-immunology-and-allergy 16/156

inactivated polio vaccine (IPV). The oral polio vaccine has not been recommended in the US since2000 due to rare cases of vaccinated individuals contracting polio from the vaccine. Children shouldreceive a series of 4 shots at the ages of: 

  2 months

  4 months

  6 – 18 months

  4 – 6 years

 Adults recommended to receive vaccination are those who: 

Page 17: Clinical Immunology and Allergy

7/25/2019 Clinical Immunology and Allergy

http://slidepdf.com/reader/full/clinical-immunology-and-allergy 17/156

 

  Expect to travel to areas where polio is endemic

   Are laboratory workers who may handle the virus

   Are healthcare workers who may treat polio patients.

IPV is contraindicated in individuals who have previously shown allergy symptoms to streptomycin,polymyxin B, neomycin, or a previous polio shot. Individuals who are moderately or severely illshould wait until they recover to receive the vaccine. 

HIV infection (choice E) is not a contraindication for polio vaccine. 

 A 48-year-old man presents to your clinic complaining of a chronic cough with fever that he has hadfor the last 2 months. On physical examination, his temperature is 37.9 C. You order a chest x-raywhich reveals a diffuse bilateral reticulonodular pattern. You decide to refer him to a thoracic

d b hi l bi i f d O i i i i f h bi h

Page 18: Clinical Immunology and Allergy

7/25/2019 Clinical Immunology and Allergy

http://slidepdf.com/reader/full/clinical-immunology-and-allergy 18/156

surgeon and a transbronchial biopsy is performed. On microscopic examination of the biopsy, thereare focal areas of inflammation containing epithelioid macrophages, Langhans giant cells, andlymphocytes. Findings are consistent with Granuloma formation with reactivation or reinfectiontuberculosis.

These findings are most typical for which of the following immunologic responses?

1)  Exaggerated cellular immune response 2)  Type II hypersensitivity 3)  Delayed type hyersensitivity 4)  CD4+ Th2 5)  Type IV hypersensitivity 

 

The correct answer is choice A.

Granulomas form as the result of a heightened cellular immune response when the body attemptsto isolate foreign substances that it cannot eliminate. The may be caused by infectious orgamisms,most commonly mycobacteria or fungi, or non-infectious means leading to conditions such assarcoidosis. The inflammatory response leading to the formation of granulomas begins withmigration of mononuclear inflammatory cells to the site. Typically, CD4 Th1 and mononuclear

phagocytes are prominent early actors. Macrophages, epitheloid cells, and giant cells follow. Thearea is frequently rimmed by T cells, primarily CD4 cells, and becomes interlaced with fibrin. 

It is thought that the clinical effects of granuloma formation are due more to the physical effects of

th l di ti l ti hit t th t i l i l f ll i th

Page 19: Clinical Immunology and Allergy

7/25/2019 Clinical Immunology and Allergy

http://slidepdf.com/reader/full/clinical-immunology-and-allergy 19/156

the granuloma disrupting normal tissue architecture than to immunological processes following therelease of immune mediators. The following table reviews the incorrect choices in this question(choices B, C and E).

Table 1. Comparison of Different Types of hypersensitivity reactions 

 A 28 year old male, who is a heavy smoker presented to your clinic with a chronic cough. He

complained of spitting up blood with coughing which developed about 1 month prior. The patient

Page 20: Clinical Immunology and Allergy

7/25/2019 Clinical Immunology and Allergy

http://slidepdf.com/reader/full/clinical-immunology-and-allergy 20/156

was otherwise healthy, but concerned about contracting tuberculosis since he travelled frequently

to countries with high incidence of tuberculosis. A tuberculin skin test was performed that read at 72

hours as 5 mm.

This is a negative result in each of the following conditions except:

1)  Health care worker  2)  Patient with Rheumatoid Arthritis 3)  Patient on corticosteroids (10mg/day x 3 weeks) 4)   A normal healthy individual 5)  Previously normal tuberculin test (within 2 years) 

 

The correct answer is choice B.

Tuberculin reaction is classified as positive based on the diameter of the induration in relation tocertain patient-specific risk factors. People with compromised immune functions will have a milderreaction to tuberculin. A positive reaction to tuberculin may be the result of a previous naturalinfection with M. tuberculosis, infection with a variety of non-tuberculosis mycobacteria (cross-reaction), or tuberculosis vaccination with a live, but weakened (attenuated) mycobacterial strain.

Induration of less than 2 mm, without blistering, is considered a negative skin test. In patients whoare immunocompromised, such as people with HIV, rheumatoid arthritis, organ transplant cases

and Crohn's disease 5 mm of induration is considered a positive skin test result Also patients who

Page 21: Clinical Immunology and Allergy

7/25/2019 Clinical Immunology and Allergy

http://slidepdf.com/reader/full/clinical-immunology-and-allergy 21/156

and Crohn s disease, 5 mm of induration is considered a positive skin test result. Also, patients whomay have contacted tuberculosis recently and persons with nodular or fibrotic changes on chest x-ray consistent with old healed TB, a 5 mm tuberculin test is considered positive. A person whoreceived a Bacille Calmette Guerin (BCG) vaccine that is administered in some countries, againsttuberculosis may also have a positive skin reaction to the TB test which is less than 10 mminduration.

In people with underlying kidney disease, diabetes, or a health-care worker (choice A) or personwith personal contact of someone with active TB, 10 mm of induration is considered a positive skintest. Moreover, 10 mm is positive in people who have arrived from high tuberculosis prevalencecountries (less than 5 years), Residents and employees of high-risk congregate settings (e.g.,prisons, nursing homes, hospitals, homeless shelters, etc.) and patients with diabetes, prolonged

corticosteroid therapy (choice C), leukemia, end-stage renal disease, chronic malabsorptionsyndromes etc.

In a healthy person whose immune system is normal, induration greater than or equal to 15 mm(choice D) is considered a positive skin test. If blisters are present (vesiculation), the test is alsoconsidered positive.

 A tuberculin test conversion is defined as an increase of 10 mm or more within a 2-year period

(choice E) regardless of age

Page 22: Clinical Immunology and Allergy

7/25/2019 Clinical Immunology and Allergy

http://slidepdf.com/reader/full/clinical-immunology-and-allergy 22/156

(choice E), regardless of age.

In summary: 

Page 23: Clinical Immunology and Allergy

7/25/2019 Clinical Immunology and Allergy

http://slidepdf.com/reader/full/clinical-immunology-and-allergy 23/156

The antibodies primarily responsible for activating the classic complement cascade include which of

Page 24: Clinical Immunology and Allergy

7/25/2019 Clinical Immunology and Allergy

http://slidepdf.com/reader/full/clinical-immunology-and-allergy 24/156

The antibodies primarily responsible for activating the classic complement cascade include which ofthe following:

1)  IgE 2)  IgD 3)  IgM, IgD 4)  IgG, IgM 

5)  IgM  

The correct answer is choice D

 Antibodies are glycoproteins in nature. Papain digestion breaks the immunoglobulin molecule fromthe hinge region, giving rise to two parts. One part is known as the Fab (fragment antigen binding).

The proteolytic fragments from the other part tends to crystallize and therefore it is known as thefragment crystallization (Fc). Fc part is made up of constant regions of the two heavy chains. Basedon the differences of amino acids in the constant region of heavy chains, immunoglobulins can bedivided into five isotypes. The antigen binding region helps antibodies to recognize and bind tovarious antigens while Fc portion can mount the necessary immune response in form of activatingvarious immune effector cells or by activating complement proteins.

Complement system is composed of a set of circulatory proteins and also the proteins attached tothe cell membranes. The function of the complement system is well described by its name as theseproteins assist or complement the antibodies in their antimicrobial activities. One pathway ofactivating the complement system of proteins is through the microbes and is called as the

Alternative pathway. Second pathway is termed as the Classical pathway and it is activated through

Page 25: Clinical Immunology and Allergy

7/25/2019 Clinical Immunology and Allergy

http://slidepdf.com/reader/full/clinical-immunology-and-allergy 25/156

 Alternative pathway. Second pathway is termed as the Classical pathway and it is activated throughthe antibodies attached to the antigens. The third pathway leading to the complement activation istermed as the lectin pathway.

The classical pathway is triggered when either IgM or certain subclasses of IgG (IgG1 and IgG3)binds to the antigen or a microbe. When the two Fc region of the two antibodies come into a closeproximity to each other, then a C1protein is recruited and attaches to the Fc regions thus forming acomplex. C1 protein becomes enzymatically active and this results in a sequence of events leadingto the activation of other proteins of the complement system like C4, C2. The activation ofcomplement system can either cause direct lysis of the foreign cells or indirectly e.g. throughopsonization.

 A 30 year old African-American was referred for immunological testing to ascertain the cause of her

meningitis following infection with Neisseria meningitidis. X-ray of the skull and sinuses was normal

and did not show any abnormal communication with the cerebrospinal fluid. Thus, a possible

immune defect was suspected.

Total and specific immunoglobulin levels were normal. Total classical pathway hemolyticcomplement activity (CH50) and alternate pathway (AP 50) were undetectable. Tests revealed an

isolated deficiency of C6, with normal levels of all other components. Half normal levels of C6 were

found in the sera of her parents and in three of four siblings. Patients with C6 complement

deficiency are at higher risk of Neisseria infection

Page 26: Clinical Immunology and Allergy

7/25/2019 Clinical Immunology and Allergy

http://slidepdf.com/reader/full/clinical-immunology-and-allergy 26/156

deficiency are at higher risk of Neisseria infection.

The complement system provides immunity through which of the following functions?

1)  Class I recognition 2)  IL-2 production 

3)   Activated T-cell response 4)  Immune complex clearing 5)   Activated B-cell response 

 

The correct answer is choice D.

The complement proteins are a group of proteins that assist in eliminating foreign pathogens and inpromoting the ensuing inflammatory reaction. Complement system helps fight infection throughlysis of foreign cells by antibody-dependent or antibody-independent pathways; opsonization oruptake of particulate antigens, including bacteria, by phagocytes; activation of inflammatoryresponses; and clearance of circulating immune complexes by cells in the liver and spleen. The

complement protein- C-reactive protein binds to the C-polysaccharide cell-wall component found ona variety of bacteria and fungi. This binding activates the complement system, resulting inincreased clearance of the pathogen either by complement-mediated lysis or by a complementmediated increase in phagocytosis. Binding of antibody to antigen on a microorganism can alsoactivate the complement system, resulting in lysis of the foreign organism. Complement includes a

collection of proteins that can perforate cell membranes.

Page 27: Clinical Immunology and Allergy

7/25/2019 Clinical Immunology and Allergy

http://slidepdf.com/reader/full/clinical-immunology-and-allergy 27/156

p p

C6 deficiencies have been shown to have a possible predisposition in African populations.Complement deficiencies of the terminal cascade proteins also predispose patients to infection, butthe clinical history of these patients is different. The terminal complement proteins are the proteinsin the cascade that form the MAC, ie, complement proteins C5-C9. These proteins are responsiblefor bactericidal killing of organisms such as N meningitidis. Class I MHC molecules are expressedby nearly all nucleated cells of vertebrates. Class I MHC recognition (choice A) is carried out by T-cell receptors that can recognize antigens only in association with the proteins of the majorhistocompatibility complex (MHC).

IL-2 (choice B) is a proliferation cytokine produced by the TH cells. TH cell activation is initiated byinteraction of the TCR-CD3 complex with a processed antigenic peptide bound to a class II MHCmolecule on the surface of an antigen-presenting cell. 

 A 20 month old male infant presents with a history of recurrent bruising, bilateral otitis media,

pneumonia, and severe eczema. Complete blood count with differential count revealsthrombocytopenia (50 × 103/mm3), low mean platelet volume (5 fL), low serum IgM, and increased

IgA and IgE levels.

What is the likely diagnosis for this patient?

Page 28: Clinical Immunology and Allergy

7/25/2019 Clinical Immunology and Allergy

http://slidepdf.com/reader/full/clinical-immunology-and-allergy 28/156

What is the likely diagnosis for this patient?

1)   Ataxia-telangiectasia 2)  DiGeorge syndrome 3)  Hyperimmunoglobulinemia E syndrome 4)  Nezelof syndrome 

5)  Wiskott-Aldrich syndrome (WAS)  

The correct answer is choice E.

WAS first manifests itself by defective responses to bacterial polysaccharides and by lower-than-

average IgM levels. Other responses and effector mechanisms are normal in the early stages of thesyndrome. As the WAS sufferer ages, there are recurrent bacterial infections and a gradual loss ofhumoral and cellular responses. The syndrome includes thrombocytopenia (lowered platelet count;the existing platelets are smaller than usual and have a short half-life), which may lead to fatalbleeding. Eczema (skin rashes) in varying degrees of severity may also occur, usually beginningaround one year of age. The defect in WAS has been mapped to the short arm of the Xchromosome and involves a cytoskeletal glycoprotein present in lymphoid cells called sialophorin

(CD43).The WAS protein is required for assembly of actin filaments required for the formation ofmicrovesicles.

 Ataxia telangiectasia (choice A) is a disease syndrome that includes deficiency of IgA and

sometimes of IgE. The syndrome is characterized by difficulty in maintaining balance (ataxia) and

Page 29: Clinical Immunology and Allergy

7/25/2019 Clinical Immunology and Allergy

http://slidepdf.com/reader/full/clinical-immunology-and-allergy 29/156

by the appearance of broken capillaries (telangiectasia) in the eyes. The primary defect appears tobe in a kinase involved in regulation of the cell cycle. The relationship between the immunedeficiency and the other defects in ataxia telangiectasia remains obscure.

In patients with DiGeorge syndrome (choice B), children are born without a thymus and thereforelack the T-cell component of the cell-mediated immune system. They are generally able to copewith infections of extracellular bacteria, but cannot effectively eliminate intracellular pathogens. Although B cells are present in normal numbers, affected individuals do not produce antibody inresponse to immunization with specific antigens. The lack of functional cell-mediated immunityresults in repeated infections with viruses, intracellular bacteria, and fungi. Even the attenuatedvirus present in a vaccine, capable of only limited growth in normal individuals can produce life-

threatening infections. It is a developmental defect, which is associated with the deletion in theembryo of a region on chromosome 22. It also manifests along with characteristic facialabnormalities, hypoparathyroidism, and congenital heart disease.

Hyper-IgE (Job syndrome) (choice C) is a primary immunodeficiency characterized by skinabcesses, recurrent pneumonia, eczema, and elevated levels of IgE. It is accompanied by facial

abnormalities and bone fragility. This multi-system disorder is autosomal dominant and has variableexpressivity. The gene for hyper IgE syndrome, or HIES, maps to chromosome 4. HIESimmunologic signs include recurrent infection and eosinophilia in addition to elevated IgE levels. Although other symptoms are similar to WAS, thrombocytopenia is missing here.

Page 30: Clinical Immunology and Allergy

7/25/2019 Clinical Immunology and Allergy

http://slidepdf.com/reader/full/clinical-immunology-and-allergy 30/156

Page 31: Clinical Immunology and Allergy

7/25/2019 Clinical Immunology and Allergy

http://slidepdf.com/reader/full/clinical-immunology-and-allergy 31/156

 

Page 32: Clinical Immunology and Allergy

7/25/2019 Clinical Immunology and Allergy

http://slidepdf.com/reader/full/clinical-immunology-and-allergy 32/156

Graft versus host disease (choice A) is a rare disorder that occurs when the immune cells fromdonated bone marrow stem cells or a blood transfusion attack the recipient. It does not produce agranulomatous reaction. 

Polyclonal B-cell activation (choice B) is a natural mode of immune response in mammals.Granulomatous reactions are based mostly upon cell-mediated immunity. 

Type I hypersensitivity (choice C) is also known as immediate or anaphylactic hypersensitivity. The

reaction usually takes 15 - 30 minutes from the time of exposure to the antigen. 

Type II hypersensitivity (choice D) is also known as cytotoxic hypersensitivity and typically affects avariety of organs and tissues. The reaction time is minutes to hours. 

Page 33: Clinical Immunology and Allergy

7/25/2019 Clinical Immunology and Allergy

http://slidepdf.com/reader/full/clinical-immunology-and-allergy 33/156

Page 34: Clinical Immunology and Allergy

7/25/2019 Clinical Immunology and Allergy

http://slidepdf.com/reader/full/clinical-immunology-and-allergy 34/156

cytokines) from mast cells. Type-1 hypersensitivity is a Th2 mediated response.

Page 35: Clinical Immunology and Allergy

7/25/2019 Clinical Immunology and Allergy

http://slidepdf.com/reader/full/clinical-immunology-and-allergy 35/156

  Type II- Antibody mediated hypersensitivity. Type II hypersensitivity ischaracterized byopsonization and phagocytosis. Prototypic disorders include auto-immune hemolyticanemias, Goodpasture Syndrome, Graves' Disease, Myesthenia gravis, Type II diabetes,pemphigus and acute rheumatoid arthritis. It is Th1 mediated.

  Type III-Immune Complex (IC) mediated hypersensitivity. It is Th1 mediated and theprototypic disorders include Systemic Lupus Erythematosus, serum sickness and someforms of glomerulonephritis. This type is composed of three phases:

1. Deposition of antigen-antibody complexes

2. Complement activation and the recruitment of inflammatory cells

3 Release of enzymes and toxic metabolites

Page 36: Clinical Immunology and Allergy

7/25/2019 Clinical Immunology and Allergy

http://slidepdf.com/reader/full/clinical-immunology-and-allergy 36/156

3. Release of enzymes and toxic metabolites

  Type IV-Cell Mediated- hypersensitivity. It is characterized by the release of TH1-typecytokines and T-cell mediated cytotoxicity. Prototypic diosrders include contact dermatitis,multiple sclerosis, type I diabetes, rheumatoid arthritis, the inflammatory bowel diseases andtuberculosis. These types of reactions are carried out by both CD4 and CD8 T cells.

Of the possible answers listed, asthma (choice A), SLE (choice D) and ulcerative colitis (choiceE) are Th2-dominant. Atopic dermatitis (choice B) appears to be the result of the sequentialactivation of Th2 followed by Th1 cells. Type I diabetes is a Type IV reaction and is notcharacterized by Th1 or Th2 dominance. 

Page 37: Clinical Immunology and Allergy

7/25/2019 Clinical Immunology and Allergy

http://slidepdf.com/reader/full/clinical-immunology-and-allergy 37/156

Page 38: Clinical Immunology and Allergy

7/25/2019 Clinical Immunology and Allergy

http://slidepdf.com/reader/full/clinical-immunology-and-allergy 38/156

Page 39: Clinical Immunology and Allergy

7/25/2019 Clinical Immunology and Allergy

http://slidepdf.com/reader/full/clinical-immunology-and-allergy 39/156

Page 40: Clinical Immunology and Allergy

7/25/2019 Clinical Immunology and Allergy

http://slidepdf.com/reader/full/clinical-immunology-and-allergy 40/156

  In Graves' disease (Choice B), stimulatory auto-antibodies form to the TSH-receptor (TSH-r).

Page 41: Clinical Immunology and Allergy

7/25/2019 Clinical Immunology and Allergy

http://slidepdf.com/reader/full/clinical-immunology-and-allergy 41/156

( ), y p ( )

  In Goodpasture's syndrome (Choice C), auto-antibodies form to a domain of the alpha-3chain of collagen typeIV. Goodpasture's syndrome is also known as anti-glomerular

basement membrane disease and is characterized by pulmonary hemorrhage andglomerulonephritis.

Therefore, the correct answer is Choice E, all of the above are Type II reactions. 

Table 1: Summary of Hypersensitivity reactions

Page 42: Clinical Immunology and Allergy

7/25/2019 Clinical Immunology and Allergy

http://slidepdf.com/reader/full/clinical-immunology-and-allergy 42/156

Page 43: Clinical Immunology and Allergy

7/25/2019 Clinical Immunology and Allergy

http://slidepdf.com/reader/full/clinical-immunology-and-allergy 43/156

Page 44: Clinical Immunology and Allergy

7/25/2019 Clinical Immunology and Allergy

http://slidepdf.com/reader/full/clinical-immunology-and-allergy 44/156

 

The induction of self-tolerance in T cells results from exposure of immature thymocytes to self

antigens and the subsequent clonal deletion of those that are self-reactive. Any tissue antigens that

are sequestered from the circulation, and are therefore not seen by the developing T cells in the

thymus, will not induce self-tolerance. Exposure of mature T cells to such normally sequestered

antigens at a later time might result in their activation.

 All of the following are a clinical manifestation of the above situation except?

1)  Neonatal myasthenia gravis 

2)   Autoimmunity after vasectomy 3)   Autoimmunity to Myelin basic protein (MBP) after head injury ) R l f l t i ft d

Page 45: Clinical Immunology and Allergy

7/25/2019 Clinical Immunology and Allergy

http://slidepdf.com/reader/full/clinical-immunology-and-allergy 45/156

4)  Release of lens protein after eye damage 5)  Systemic Lupus Erythematosus 

 

The correct answer is choice A.

 As stated above, any self antigen that the developing T cells do not encounter during developmentwill be treated as non-self; hence the choices B, C and D are correct.

Myelin basic protein (MBP) is an example of an antigen normally sequestered from the immunesystem, by the blood-brain barrier. However, trauma to tissues following either an accident or a viralor bacterial infection can also release sequestered MBP (choice C) into the circulation resulting inautoimmunity.

Sperm arise late in development and are sequestered from the circulation. However, after a

vasectomy, some sperm antigens are released into the circulation and can induce auto-antibodyformation in some men (choice B).

Similarly, the release of lens protein after eye damage or of heart-muscle antigens after myocardialinfarction has been shown to lead on occasion to the formation of auto-antibodies (choice D).

Page 46: Clinical Immunology and Allergy

7/25/2019 Clinical Immunology and Allergy

http://slidepdf.com/reader/full/clinical-immunology-and-allergy 46/156

In patients with myasthenia gravis (choice A) auto-antibodies that bind to the acetylcholinereceptors on the motor end-plates of muscles are produced, blocking the normal binding of

acetylcholine and also inducing complement mediated lysis of the cells. As a result, a progressiveweakening of the skeletal muscles occurs. Ultimately, the antibodies destroy the cells bearing thereceptors. Although this is an autoimmune disorder, acetylcholine receptors are not sequestered inthe body. 

 A 3 month old male infant was admitted for investigation of repeated infections and failure to thrive.

He was born full term and APGAR scores were appropriate. He was not given BCG at birth. The

infant was healthy till 2 months of age when he was administered antibiotics for a chest infection,

following which he developed diarrhea and re-infection of his chest occurred. Due to this, further

immunizations were postponed. There were no rashes or lymphadenopathy but his liver was

palpable just below the right costal margin. He had slight tachycardia and tachypnoea on

examination. Investigations showed a marked deficiency of T cells with normal numbers of B cells,

with no immunoglobulin production. He had a T- B+ form of severe combined immune deficiency.

What would be the normal course of therapy for the patient?

1)  Treatment with IFN-γ 2)  Bone marrow transplant 3)   Administration of immunoglobulins 

Page 47: Clinical Immunology and Allergy

7/25/2019 Clinical Immunology and Allergy

http://slidepdf.com/reader/full/clinical-immunology-and-allergy 47/156

incidence of infections. Also, the infections that are contracted are less severe.

Page 48: Clinical Immunology and Allergy

7/25/2019 Clinical Immunology and Allergy

http://slidepdf.com/reader/full/clinical-immunology-and-allergy 48/156

 Administration of immunoglobulins (choice C) is helpful in disorders where antibodies are absentsuch as specific IgA and IgG deficiency, common variable immunodeficiency (CVI),agammaglobulinemia etc.

Recombinant IL-2 (choice D) may help to restore immune function in diseases where immunityagainst a virus is required. It is generally used in therapy in AIDS patients and in certain cancers.

 An 8-year old boy presents to the emergency department in respiratory distress. His symptoms

began within 15 minutes after he ate a chocolate chip cookie, and he has had similar reactions aftereating foods containing peanuts and other ingredients. Physical examination reveals diffuseurticaria, angioedema and bilateral wheezing. The patient is treated with epinephrine,antihistamines and oral corticosteroids, and he recovers.

What type of hypersensitivity reaction has this patient experienced?1)  Type I 2)  Type II 

3)  Type III 4)  Type Iva 5)  Type Ivb 6)  MIxed Type hypersensitivity 

 

Page 49: Clinical Immunology and Allergy

7/25/2019 Clinical Immunology and Allergy

http://slidepdf.com/reader/full/clinical-immunology-and-allergy 49/156

Page 50: Clinical Immunology and Allergy

7/25/2019 Clinical Immunology and Allergy

http://slidepdf.com/reader/full/clinical-immunology-and-allergy 50/156

Page 51: Clinical Immunology and Allergy

7/25/2019 Clinical Immunology and Allergy

http://slidepdf.com/reader/full/clinical-immunology-and-allergy 51/156

 

 A 25-year-old man presents in your urgent care center with a high fever and cough productive of

yellowish sputum that he has had for approximately 24 hours. Laboratory tests reveal a serum IgA

of 22 mg/dL, IgG of 175 mg/dL, and IgM of 40 mg/dL. Auscultation of his chest reveals a few

crackles in both lung bases. A chest radiograph shows bilateral patchy pulmonary infiltrates.

Which of the following inflammatory cell types will most likely be increased in his sputum specimen?

1)  Macrophage 2)  Mast cell 

3)  Neutrophil 4)  Lymphocyte 5) Langhans giant cell

Page 52: Clinical Immunology and Allergy

7/25/2019 Clinical Immunology and Allergy

http://slidepdf.com/reader/full/clinical-immunology-and-allergy 52/156

5)  Langhans giant cell  

The correct answer is choice C.

To diagnose pneumonia, health care providers rely on a patient's symptoms and findings fromphysical examination. Information from a chest X-ray, blood tests, and sputum cultures are helpfulto confirm the diagnosis. Bacterial infections like pneumonia promote an acute inflammatoryresponse dominated by neutrophils, which gives the sputum the yellowish, purulent appearance.

In addition to a cough that produces greenish or yellow sputum, people with infectious pneumoniaoften have and a high fever that may be accompanied by shaking chills. Shortness of breath is alsocommon, as is pleuritic chest pain, a sharp or stabbing pain, either felt or worse during deepbreaths or coughs. The decreased serum immunoglobulins at his age suggest common variableimmunodeficiency, which puts him at risk for bacterial infections such as pneumonia.

The role of macrophages (choice A) is to phagocytose cellular debris and pathogens either as

stationary or as mobile cells and are mainly part of the process of resolution of inflammation.

Page 53: Clinical Immunology and Allergy

7/25/2019 Clinical Immunology and Allergy

http://slidepdf.com/reader/full/clinical-immunology-and-allergy 53/156

Mast cells (choice B) degranulate and release vasoactive substances as part of type 1hypersensitivity reactions.

Lymphocytes (choice D) are the 'round cells' of chronic inflammatory infiltrates.

Langhans giant cells (choice E) are typically seen with infectious granulomas.

 A 47-year-old man presents in your office with a variety of complaints. Over the past year he has

developed pain and swelling around the small joints of his hands and feet. Additionally, he has

chronic diarrhea and has lost significant weight - approximately 3.5 kg. His chart reveals that the

patient has had several upper respiratory tract infections during this time, including such organisms

as Staphylococcus aureus, Streptococcus pneumoniae, and Pseudomonas aeruginosa cultured. A

stool specimen is found to have Giardia lamblia cysts. Laboratory studies show a hemoglobin of 8.8

g/dL with a reticulocyte count of 3.2%. His serum urea nitrogen is 19 mg/dL with creatinine 1.0

mg/dL. His antinuclear antibody test is negative. Quantitative serum immunoglobulins show IgA 70

mg/dL, IgG 303 mg/dL, and IgM 64 mg/dL. By flow cytometry there are normal numbers of T cells

Page 54: Clinical Immunology and Allergy

7/25/2019 Clinical Immunology and Allergy

http://slidepdf.com/reader/full/clinical-immunology-and-allergy 54/156

serum levels of IgG and IgA are characteristic, approximately 50% of patients with the deficiencyalso have diminished serum IgM levels and T-lymphocyte dysfunction. 

Page 55: Clinical Immunology and Allergy

7/25/2019 Clinical Immunology and Allergy

http://slidepdf.com/reader/full/clinical-immunology-and-allergy 55/156

Patients come to medical attention due to infectious diseases at the time of onset, the mostcommon being otitis media, diarrhea, pneumonia, and sinusitis. Almost all have acute and recurrent

infections. Persistent diarrhea and malabsorption caused by Giardia lamblia infection also occur inpatients with CVID, leading to significant weight loss and, in children, failure to thrive. 

The mainstay of treatment for CVID is Ig replacement therapy with antimicrobial therapy initiated atthe first sign of infection. Although expensive, Ig replacement therapy stops the cycle of recurrentinfections. 

Bruton's disease (choice A) manifests early in childhood with a marked reduction in all serumglobulins. 

Polymyositis (choice C) is characterized by the presence of Anti Jo antibodies in >65% of patients.Elevated serum creatine kinase is also characteristic, but not specific to polymyositis. 

The bacterial infections suggest hypogammaglobulinemia, not a primary or secondary T cell defectas could occur with HIV infection (choice D). 

Page 56: Clinical Immunology and Allergy

7/25/2019 Clinical Immunology and Allergy

http://slidepdf.com/reader/full/clinical-immunology-and-allergy 56/156

Though persons with diabetes mellitus (choice E) are prone to bacterial infections, they not likely to

get a Giardia diarrhea or hemolytic anemia in conjunction. 

 A 38-year-old man arrives in the emergency room following a motor vehicle accident with multiple

lacerations to the lower left leg. He has significant blood loss, requiring transfusion of blood

products. In taking his oral medical history, it is revealed that he has frequent bouts of diarrhea of

unknown etiology and has had a significant number of respiratory tract infections over the past ten

years. During the transfusion, he has an anaphylactic transfusion reaction.

Which of the following diseases is he most likely to have?

1)  Selective IgA deficiency 2)  Graft versus host disease 3)  System lupus erythematosus 

4)  Niemann-Pick disease 5)   Amaloidosis 

 

The correct answer is choice A.

Page 57: Clinical Immunology and Allergy

7/25/2019 Clinical Immunology and Allergy

http://slidepdf.com/reader/full/clinical-immunology-and-allergy 57/156

 Approximately one out of every 600 people has selective IgA deficiency. Among those with thisdisease, people of European ancestry greatly outnumber those of other ethnic groups. People with

this deficiency lack IgA, a type of antibody that protects against infections of the mucousmembranes lining the mouth, airways, and digestive tract. IgA deficiency is caused by faulty B cells.While patients have normal numbers of B cells, these cells do not mature into normal IgA-producingcells. Scientists do not know the exact cause or causes for these immature B cells. Sometimesclusters of cases occur in families. People with IgA-deficiency are more likely than the generalpopulation to be related to someone with combined variable immunodeficiency, another form ofimmune deficiency. 

Many people with IgA-deficiency are healthy, with no more than the usual number of infections.Those who do have symptoms typically have recurring ear, sinus, or lung infections that may notrespond to regular treatment with antibiotics. People with IgA-deficiency are likely to have otherproblems, such as allergies, asthma, chronic diarrhea, and autoimmune diseases. 

IgA deficiency is diagnosed by doing tests to measure the amount of total Ig in the blood as well asIgG2. There is no specific treatment for selective IgA deficiency. Bacterial infections are treated with

antibiotics. 

Page 58: Clinical Immunology and Allergy

7/25/2019 Clinical Immunology and Allergy

http://slidepdf.com/reader/full/clinical-immunology-and-allergy 58/156

Graft versus host disease (choice B) is rare, but possible from a blood transfusion, however theresults take weeks to manifest. 

In systemic lupus erythematosus (choice C), there is more likely to be a polyclonal gammopathy.There is no tendency to type 1 hypersensitivity reactions. 

Niemann-Pick disease (choice D) is a metabolic disease that is typically fatal by 18 months. Thereis an adult form of Niemann-Pick disease, but immunologic reactions are not part of this disease. 

 Amyloidosis (choice E) occurs when amyloid proteins build up in your organs and is not marked byhypersensitivity reactions. 

 A 77-year-old woman was admitted because of sudden onset of massive angioedema of her

tongue associated with laryngeal stridor. She was treated with intravenous hydrocortisone. This

was her fifth such episode. She had also experienced an anaphylactoid attack two months earlier.

Page 59: Clinical Immunology and Allergy

7/25/2019 Clinical Immunology and Allergy

http://slidepdf.com/reader/full/clinical-immunology-and-allergy 59/156

starting the medicine. 

Page 60: Clinical Immunology and Allergy

7/25/2019 Clinical Immunology and Allergy

http://slidepdf.com/reader/full/clinical-immunology-and-allergy 60/156

 Angioedema is the swelling of deep dermis, subcutaneous or submucosal tissue due to vascularleakage. Clinically, angioedema is usually nonpitting and nonpruritic sometimes with mild pain.Laryngeal swelling however can be life-threatening.

The variable portion of the immunoglobulin is represented by which of the following:1)  The antigenic end of the heavy and light chains 

2)  The antigenic ends of the light chains 3)  The constant end 4)   A and B 5) All of the above

Page 61: Clinical Immunology and Allergy

7/25/2019 Clinical Immunology and Allergy

http://slidepdf.com/reader/full/clinical-immunology-and-allergy 61/156

5)   All of the above  

The correct answer is choice A

B cells harbor receptors termed as the B cell receptors (BCR) which enable B cells to bind andrespond to epitopes (the part of the antigen recognized by BCR) in presence of specific signals. Activation of B cells through their BCRs leads to the differentiation of B cells into plasma cells whichare capable of producing soluble forms of the BCR, commonly known as Antibodies. Unlike the Tcell receptors which remain attached to the T cells, soluble antibodies generated from B cells arecapable of performing their functions throughout the whole body away from their site of production.

 Antibodies are glycoproteins. The antibody molecule is a Y-shaped structure consisting of fourchains, two light and two heavy chains. Both of the chains contain one Variable (V) and one or

more Constant (C) regions. The first amino acids present at the amino terminal of antibodies, variesin all antibodies. Therefore these are known as the Variable regions. No two B cells are likely toproduce the antibodies with the same variable regions. 

This variability is most pronounced in the region known as the Hypervariable region. Threehypervariable regions of the light and heavy chains form the Antigen binding site where an epitopecan be recognized and attached These are also known as the Complementarity determining

Page 62: Clinical Immunology and Allergy

7/25/2019 Clinical Immunology and Allergy

http://slidepdf.com/reader/full/clinical-immunology-and-allergy 62/156

can be recognized and attached. These are also known as the Complementarity determiningregions. The fragment of antibody consisting of the variable (V) and constant domains of the lightand heavy chain are required to bind to the antigens and are therefore known as the Fab (fragmentantigen binding). 

 A 32 year old G3P2 pregnant (22 weeks) woman presented to the local hospital case room and

complained about her fetus not growing. The patient had an uneventful first pregnancy and had a

normal 5 year old son. Her second pregnancy ended with dead hydropic fetus at 24 weeks. The

fetus had a very low hemoglobin (70g/l), indicating hemolytic disease of the newborn. The woman

was Rhesus (Rh) negative while the fetus was found to be Rhesus D positive. Rh alloimmunization

was suspected.

Which antibody subtype is responsible for Rh alloimmunization?

1)  IgA 2)  IgG 3)  IgM 4)  IgD 5)  IgE 

 

The correct answer is choice B

Page 63: Clinical Immunology and Allergy

7/25/2019 Clinical Immunology and Allergy

http://slidepdf.com/reader/full/clinical-immunology-and-allergy 63/156

Hemolytic disease of the newborn (HND) or Erythroblastosis fetalis is caused when blood antigenincompatibility occurs between the fetus and the mother, usually due to Rh alloimmunization.During pregnancy, fetal red blood cells are separated from the mother‟s circulation by the

trophoblast. During her first pregnancy with an Rh+ fetus, an Rh – woman is usually not exposed toenough fetal red blood cells to activate her Rh-specific B cells. At the time of delivery, however,separation of the placenta from the uterine wall allows larger amounts of fetal umbilical-cord bloodto enter the mother‟s circulation. These fetal red blood cells activate Rh-specific B cells, resulting inproduction of Rh-specific plasma cells and memory B cells in the mother. The secreted IgMantibody clears the Rh+ fetal red cells from the mother‟s circulation, but the memory cells remain. Activation of these memory cells in a subsequent pregnancy with an Rh+ fetus results in theformation of IgG anti-Rh antibodies, which cross the placenta and damage the fetal red blood

cells.This fetal disease ranges from mild to very severe, and fetal death from heart failure (hydropsfetalis) can occur.

Pregnancies at risk of HND are those in which an Rh D-negative mother becomes pregnant with anRhD-positive child (the child having inherited the D antigen from the father). HDN can also be

caused by an incompatibility of the ABO blood group. It arises when a mother with blood type Obecomes pregnant with a fetus with a different blood type (type A, B, or AB). The mother's serumcontains naturally occurring anti-A and anti-B, which tend to be of the IgG class and can thereforecross the placenta and hemolyse fetal RBCs.

 

HDN due to ABO incompatibility is usually less severe than Rh incompatibility because fetal RBCsexpress less of the ABO blood group antigens compared with adult levels In addition in contrast to

Page 64: Clinical Immunology and Allergy

7/25/2019 Clinical Immunology and Allergy

http://slidepdf.com/reader/full/clinical-immunology-and-allergy 64/156

express less of the ABO blood group antigens compared with adult levels. In addition, in contrast tothe Rh antigens, the ABO blood group antigens are expressed by a variety of fetal (and adult)tissues, reducing the chances of anti-A and anti-B binding their target antigens on the fetal RBCs.Only IgG antibodies can cross the placenta. IgA antibodies (choice A) are involved in the mucosal

immunity and do not cross the placenta.

IgM and IgD antibodies (choice C and D respectively) are expressed on the surface of B cells in theearly stages of B cell mediated (humoral) immunity, are also unable to cross the placenta.

IgE (choice E) antibodies cause degranulation of mast cells to release histamines, and areimplicated in type 1 hypersensitivity reactions. IgE immunoglobulins cannot cross the placentalbarrier.

Hypersensitivity refers to undesirable (damaging, discomfort producing and sometimes fatal)

reactions produced by the normal immune system. Hypersensitivity reactions require a pre-sensitized (immune) state of the host. Hypersensitivity reactions can be divided into four types: typeI, type II, type III and type IV, based on the mechanisms involved and time taken for the reaction.Frequently, a particular clinical condition (disease) may involve more than one type of reaction.

 An appropriate, useful type I hypersensitivity response of the immune system, accompanied byeosinophilia, would most likely be directed against which of the following?

1)  Flavivirus 2) E. coli O157

Page 65: Clinical Immunology and Allergy

7/25/2019 Clinical Immunology and Allergy

http://slidepdf.com/reader/full/clinical-immunology-and-allergy 65/156

2)  E. coli O157 3)  Spirochetes 4)  Liver flukes 5)  Staph. Aureus 

 

The correct answer is choice D

Liver flukes are a type of parasitic worm or helminth, a multi-cellular organism that infects both

humans and animals. While infection frequently cases morbidity and occasionally may be fatal,helminthes have evolved highly complex, immunomodulatory mechanisms that allow them tocoexist with their host. 

The immune response to parasitic infections has been termed „skewed Th2”. Infection is followed

by eosinophilia and production of IgE. The specific immune response is determined by the locationof the infection. When the parasites have infected a lumen, for example, the gut, type 1hyerpsensitivity and antibody dependent cellular cytotoxic (ADCC) mechanisms block the larvae‟smigration into the gut and initiate elimination of the parasite. Muscle contractions and mucus

secretion in the gut expel adult helminthes from the GI system. 

Page 66: Clinical Immunology and Allergy

7/25/2019 Clinical Immunology and Allergy

http://slidepdf.com/reader/full/clinical-immunology-and-allergy 66/156

Tissue infections are primarily mediated by larvae. Antibodies, including IgE, IgG, and IgA, bind tothe infecting organism and trigger destruction by cells that have receptors to the Fc portion of theantibody. In some cases, the resulting immune reaction culminates in the development of a

granuloma which surrounds and isolates the parasite, stopping its migration through the tissue. 

Epidemiologic studies have shown an inverse relationship between parasitic infections and theprevalence of allergy and autoimmune disease. These conclusions may seem counter-intuitive. Allergic symptoms are triggered by the IgE-mediated degranulation of mast cells and basophils andthe consequent release of histamine. Nevertheless, experimental data have shown that parasitic

infections can inhibit allergic reactions and that treatment of parasitic infections can result inincreased cutaneous response to allergens. 

Immune responses to viral or bacterial infections are primarily mediated by IgG. 

Type I Hypersensitivity Review

It is also known as immediate or anaphylactic hypersensitivity. The reaction may involve skin

Page 67: Clinical Immunology and Allergy

7/25/2019 Clinical Immunology and Allergy

http://slidepdf.com/reader/full/clinical-immunology-and-allergy 67/156

 

Diagnostic tests for immediate hypersensitivity include skin (prick and intradermal) tests,measurement of total IgE and specific IgE antibodies against the suspected allergens. Total IgE

Page 68: Clinical Immunology and Allergy

7/25/2019 Clinical Immunology and Allergy

http://slidepdf.com/reader/full/clinical-immunology-and-allergy 68/156

and specific IgE antibodies are measured by a modification of enzyme immunoassay (ELISA).Increased IgE levels are indicative of atopic condition, although IgE may be elevated in some nonatopic diseases (e.g., myelomas, helminthic infection, etc.).

Symptomatic treatment is achieved with antihistamines which block histamine receptors.Chromolyn sodium inhibits mast cell degranulation, probably, by inhibiting Ca++ influx. Late onsetallergic symptoms, particularly bronchoconstriction which is mediated by leukotrienes are treatedwith leukotriene receptor blockers (Singulair, Accolate) or inhibitors of cyclooxygenase pathway(Zileutoin). Symptomatic, although short term relief from bronchoconstriction is provided by

bronchodilators (inhalants) such as isoproterenol derivatives (Terbutaline, Albuterol). Thophyllineelevates cAMP by inhibiting cAMP-phosphodiesterase and inhibits intracellular Ca++ release is alsoused to relieve bronchopulmonary symptoms. 

There appears to be a genetic predisposition for atopic diseases and there is evidence for HLA (A2)

association. 

Hyposensitization (immunotherapy or desensitization) is another treatment modality which is

successful in a number of allergies, particularly to insect venoms and, to some extent, pollens. Themechanism is not clear, but there is a correlation between appearance of IgG (blocking) antibodiesand relief from symptoms. Suppressor T cells that specifically inhibit IgE antibodies may play a role. 

Page 69: Clinical Immunology and Allergy

7/25/2019 Clinical Immunology and Allergy

http://slidepdf.com/reader/full/clinical-immunology-and-allergy 69/156

Table 1: Comparison of Different Types of hypersensitivity 

Cytokines are any of a number of small proteins that are secreted by specific cells of the immune

system and glial cells, which carry signals locally between cells, and thus have an effect on other

cells They are a category of signaling molecules that are used extensively in cellular

Page 70: Clinical Immunology and Allergy

7/25/2019 Clinical Immunology and Allergy

http://slidepdf.com/reader/full/clinical-immunology-and-allergy 70/156

cells. They are a category of signaling molecules that are used extensively in cellular

communication. They are proteins, peptides, or glycoproteins. The term cytokine encompasses a

large and diverse family of polypeptide regulators that are produced widely throughout the body by

cells of diverse embryological origin.

Which of the following is a cytokine produced by T-cells that acts mainly to activate B-cells to

proliferate:

1)  Interleukin 1 2)  Interleukin 2 

3)  Interleukin 4 4)  Interleukin 10 5)  Interferon gamma 

 

The correct answer is choice B

The functions of the cytokines in immune regulation are complex and still in the process of beingdefined. Many if not all of the cytokines have multiple functions and many are secreted by more

than one cell type. 

IL2 i ti t th l tt it i d d b Th1 ll d t t ti l t b th T d B ll

Page 71: Clinical Immunology and Allergy

7/25/2019 Clinical Immunology and Allergy

http://slidepdf.com/reader/full/clinical-immunology-and-allergy 71/156

IL2 is an exception to the latter; it is produced by Th1 cells and acts to stimulate both T and B cellsand to induce a number of cytokines and adhesion molecules. 

IL1 is made by B cells, dendritic cells, macrophages, and other immune cells. It acts across a broadspectrum, stimulating T and B cells and resulting in production of a number of cytokines. 

IL4 is produced by a number of types of T cells as well as mast cells. It induces Th2 cells and,

consequently, isotype switching to IgE. 

IL10 is made by T and B cells, macrophages, and monocytes and acts to inhibit T cell proliferationand IL2 secretion. It down-regulates MHC II expression and also stimulates B cell differentiation. 

Interferon gamma is primarily antiviral. It is made by T cells and natural killer (NK) cells. In additionto inhibiting viral replication, it increases expression of MHC I and also MHC II. It activates

macrophages and down-regulates the Th2 response. 

Page 72: Clinical Immunology and Allergy

7/25/2019 Clinical Immunology and Allergy

http://slidepdf.com/reader/full/clinical-immunology-and-allergy 72/156

Page 73: Clinical Immunology and Allergy

7/25/2019 Clinical Immunology and Allergy

http://slidepdf.com/reader/full/clinical-immunology-and-allergy 73/156

 A 4-year-old boy presents to his primary care physician for a well-child examination and routine

immunization. In the time between his first measles-mumps-rubella (MMR) vaccine and now, he

has developed severe atopic dermatitis. Evaluation by an allergist/immunologist revealed IgE-

Page 74: Clinical Immunology and Allergy

7/25/2019 Clinical Immunology and Allergy

http://slidepdf.com/reader/full/clinical-immunology-and-allergy 74/156

as de e oped se e e a op c de a s a ua o by a a e g s / u o og s e ea ed g

mediated allergy to egg, soy, and peanut. He has had anaphylaxis after eating peanuts, and eating

eggs results in urticaria and worsening of his atopic dermatitis. His mother read that MMR vaccine

is produced from eggs and is concerned about a possible reaction.

What is the next course of action for this patient?

1)  Proceed with the vaccination 2)  Refer the patient for desensitization to egg protein and then proceed with

immunization 

3)  Delay vaccination because of the history of documented egg allergy 4)  Repeat skin prick testing and, if negative, proceed with immunization 5)  Use gradually increasing doses of the vaccine to decrease the risk of a reaction 

 

The correct answer is choice A

The instances of food allergy are most frequent in childhood and mostly such conditions areimproved with age. MMR is a triple viral vaccination for measles, mumps and rubella. The

vaccination of measles and mumps are the live attenuated viruses which are grown in fibroblastsderived from the chicken embryo while attenuated rubella virus is obtained from diploid humanfibroblasts. Despite of all the controversies regarding MMR being the cause of egg allergy, the Food Allergy Committee of Spanish Society of Clinical Immunology and Pediatric Allergy has declaredMMR vaccination to be safe for children with egg allergy except with the patients with severe

Page 75: Clinical Immunology and Allergy

7/25/2019 Clinical Immunology and Allergy

http://slidepdf.com/reader/full/clinical-immunology-and-allergy 75/156

MMR vaccination to be safe for children with egg allergy except with the patients with severeanaphylactic reactions after egg ingestion are recommended to receive vaccination in theirreference hospital.

 According to one report, MMR vaccine contains only negligible amount of egg proteins (0.5 to 1 ngof ovoalbumin per 0.5 ml dose of vaccine) while compared to this amount there is a higher amountof gelatin (14.5 mg) and neomycin sulfate (25ug) contained in a single dose of vaccine andtherefore these allergens are the thought to be the likely candidates for the anaphylactic reactionsafter the MMR vaccination.

 A 54-year-old male was diagnosed with IgG1 multiple myeloma. The patient had rapidly

progressive refractory disease with the serum paraprotein rising 6 –10 g/l each month, having

progressed following multiple previous chemotherapeutic regimes and autologous stem cell

transplantation. Finally immunotherapeutic strategies were used to treat this advanced refractory

myeloma with a series of four vaccinations using autologous idiotype-protein pulsed dendritic cellscombined with adjuvant GM-CSF. The vaccinations were well tolerated with a mild fever post-

vaccination. An idiotype-specific T-cell proliferative response developed which was associated with

the production of gamma-interferon, indicating a TH-1-like response. The patient also developed

anti-idiotype IgM antibodies although no idiotype-specific cytotoxic T-cell response could be

demonstrated.

Which one of the following statements is false regarding this immunotherapeutic strategy used in

Page 76: Clinical Immunology and Allergy

7/25/2019 Clinical Immunology and Allergy

http://slidepdf.com/reader/full/clinical-immunology-and-allergy 76/156

g g g p gy

this case?

1)  Idiotypes are created by the hypervariable regions on the Fab fragment of IG's 2)  The epitopes that are shared within a species are called idiotypic determinants 3)  The idiotype of the immunoglobulin produced by myeloma cells may function as a

tumour-specific antigen 4)  Dendritic cells are powerful antigen-presenting cells, pivotal in initiating immune

response 5)  Vaccination in cancer patients with tumour-antigen loaded dendritic cells may result in

clinical response 

 

The correct answer is choice B.

The epitopes that are shared within a species and differ between species are called isotypicdeterminants (not idiotypic determinants) . Differences in antibodies among individuals of samespecies are called the allotypic determinants. Isotypic and allotypic determinants are associatedwith the constant regions of heavy and light chain genes of immunoglobulin (Ig). Hence option B is

incorrect. 

Idiotypic determinants confer an immunoglobulin (antibody) molecule an antigenic individuality that

Page 77: Clinical Immunology and Allergy

7/25/2019 Clinical Immunology and Allergy

http://slidepdf.com/reader/full/clinical-immunology-and-allergy 77/156

Idiotypic determinants confer an immunoglobulin (antibody) molecule an antigenic individuality thatis analogous to the individuality of the molecule's antibody activity. Unique antigenic determinantsare present on individual antibody molecules. Idiotypes are the antigenic determinants created by

the hypervariable regions on the Fab fragment of the Ig molecules (option A) 

The malignant cells in myeloma rearrange their immunoglobulin genes and produce a monoclonalimmunoglobulin which is secreted into the serum. This immunoglobulin is unique to the malignantclone and presents antigenic determinants, or idiotypes, within their variable regions which mayfunction as a tumour-specific antigen (option C). Therefore these idiotypes represent a potential

target for immunotherapy utilizing these proteins to generate an anti-idiotype immune response asin this case. 

Dendritic cells (DC) are powerful antigen-presenting cells which are pivotal in initiating the immuneresponse (option D). They are capable of activating naive T lymphocytes through the presentationof antigenic peptides bound to major histocompatibility complexes (MHC) in the presence ofappropriate co-stimulatory signals. Recently, vaccination of patients with tumour-antigen loaded DChas resulted in clinical responses in B-cell lymphoma and melanoma (option E) 

 A 7 month old female infant presents to your office with a complaint of chronic cough for the past 3

months and post-tussive vomiting for past 3 weeks. The infant was born full-term with no neonatal

complications. There is no history of asthma or recurrent infection in the family. Physical

examination reveals a small, emaciated infant (weight at < 5th percentile, height at 10th percentile),

Page 78: Clinical Immunology and Allergy

7/25/2019 Clinical Immunology and Allergy

http://slidepdf.com/reader/full/clinical-immunology-and-allergy 78/156

, ( g p , g p ),

extensive oral thrush, generalized lymphadenopathy (including cervical, axillary, inguinal, and

suboccipital), bilateral intercostal and subcostal retraction, bilateral crackles, hepatosplenomegaly,

and extensive diaper rash. Serum IgG, IgA, and IgM levels are all above normal.

What is the most likely diagnosis for this patient?

1)   Adhesion molecule deficiency 2)   Asthma 3)  Bruton‟s agammaglobulinemia 

4)  HIV infection 5)  Severe combined immunodeficiency (SCID) 

 

The correct answer is choice D.

Presence of opportunistic infections may point to an immunodeficiency. Failure to thrive,generalized lymphadenopathy and high immunoglobulin levels could be an indication of HIVinfection.

 

The classic descriptions of LAD (choice A) (leukocyte adhesion deficiency) includes recurrentbacterial infections, defects in neutrophil adhesion, and a delay in umbilical cord sloughing. The

Page 79: Clinical Immunology and Allergy

7/25/2019 Clinical Immunology and Allergy

http://slidepdf.com/reader/full/clinical-immunology-and-allergy 79/156

, p , y g gdefects in adhesion result in poor neutrophil chemotaxis and phagocytosis. A WBC differential willreveal extremely elevated levels of neutrophils (of the order of 6-10x normal) because they areunable to leave the blood vessels. Specific diagnosis is made through monoclonal antibody testingfor CR3, one of the three complete proteins which fail to form properly as a result of β-2 integrinsubunit deficiency.

 Asthma patients (choice B) show high IgE levels, high eosinophils, along with bronchospasm andrespiratory distress all of which are absent in this case.

X-linked agammaglobulinaemia (Bruton's disease) is characterized bypanhypogammaglobulinaemia with absent antibody production and the absence of mature Blymphocytes. Since, in this case, the antibody titres are all above normal, it cannot be Bruton‟sdisease (X-linked agammaglobulinaemia) (choice C).

Severe combined immune deficiency, (SCID) (choice E) is characterized by defects in either B or Tcells or both with no immunoglobulin production. Thus, although patients with SCID suffer from

recurrent infections, their immunoglobulin levels are not high as in this case.

 A 22 year old caucasian woman presents to your outpatient clinic with a 7 day history of a sinusinfection. When questioned, she reveals a history of recurrent, respiratory infections and allergy

Page 80: Clinical Immunology and Allergy

7/25/2019 Clinical Immunology and Allergy

http://slidepdf.com/reader/full/clinical-immunology-and-allergy 80/156

symptoms that occur regularly each spring. She is otherwise healthy and does not take any regularmedication except birth control pills. She has no known drug allergies but does complain ofenvironmental allergies. In terms of family history, her younger sister has been diagnosed with juvenile rheumatoid arthritis.

Which of the following can be suspected pending completion of laboratory analyses?1)  IgG deficiency 2)  IgA deficiency 3)  Deficiency of antibodies to polysaccharide antigens 4)  IgG3 deficiency 

5)  IgM deficiency  

The correct answer is choice B.

IgA is the most common antibody in the body and is found both in serum and in the secretions ofthe respiratory, gastrointestinal and genitourinary systems.

Specific deficiency of IgA is the most common primary immunodeficiency. The prevalence of IgAdeficiency is influenced by ethnicity; studies in US Caucasians have shown prevalence that variesbetween 1:233 to 1:3,000. Asians have a lower prevalence of the condition with studies showingprevalence of 1:2,600 to 1:18,500. Prevalence among individuals in the Arabian peninsula has

Page 81: Clinical Immunology and Allergy

7/25/2019 Clinical Immunology and Allergy

http://slidepdf.com/reader/full/clinical-immunology-and-allergy 81/156

been shown to be 1:143. Transmission of the deficiency is autosomal dominant with incompletepenetrance.

Many IgA-deficient individuals exhibit no symptoms. However, some IgA-deficient patientsexperience recurrent infections, primarily sinopulmonary and gastrointestinal, as well as allergiesand autoimmune disease. Recurrent sinopulmonary infections are the most common condition. GIconditions experienced by the patient deficient in IgA include infections, malabsorption, lactoseintolerance, celiac disease, and ulcerative colitis. It is thought that these patients‟ impaired mucosal

clearance may predispose them or act as a causative factor in their developing food and inhalantallergies. Studies on the relative prevalence of allergy among IgA deficient patients vary with mostfinding a higher risk of allergy in this population. Individuals deficient in IgA have an elevated risk ofdeveloping a range of autoimmune disorders; further, the risk of autoimmune disease is elevated infirst-degree relatives of these patients.

Because IgA-deficient patients may develop antibodies to IgA, blood transfusions are a criticalconcern. Most sources recommend that patients wear a medic-alert bracelet and that they receivewashed red cells to avoid a possible anaphylactic reaction. Similarly, patients should avoid IVIG;they do not need the IgG and residual IgA in the formulation may trigger an immune reaction.

Page 82: Clinical Immunology and Allergy

7/25/2019 Clinical Immunology and Allergy

http://slidepdf.com/reader/full/clinical-immunology-and-allergy 82/156

Page 83: Clinical Immunology and Allergy

7/25/2019 Clinical Immunology and Allergy

http://slidepdf.com/reader/full/clinical-immunology-and-allergy 83/156

Page 84: Clinical Immunology and Allergy

7/25/2019 Clinical Immunology and Allergy

http://slidepdf.com/reader/full/clinical-immunology-and-allergy 84/156

hydroxylase (21OHAb) can be found in high proportion (80 –90%) of patients with Addison's diseaseat diagnosis. Because aldosterone is absent, hyponatremia, with low chloride and hyperkalemia areoften present. Hyponatremia is the most common finding and occurs in 90% of patients.Hyperkalemia is found in 60-70% of patients. For this, reason IV fluids are administered along withhydrocortisone, to balance the electrolytes. However, this alone cannot form the treatment regime(Ch i D)

Page 85: Clinical Immunology and Allergy

7/25/2019 Clinical Immunology and Allergy

http://slidepdf.com/reader/full/clinical-immunology-and-allergy 85/156

(Choice D).

 As explained in the table, introduced in 1960s the „short synacthen test‟ remains the gold standardtest for the biochemical diagnosis of Addison's disease. Cortrosyn is a synthetic corticotropin, whichis intravenously administered with a dose of 350 mg. Serum cortisol levels are measured fromblood samples drawn after 30 and 60 minutes. Peak serum cortisol levels greater than 635 nmol/lexclude the diagnosis of adrenal insufficiency because the response to stimulation is consideredadequate at this level. Cortisol levels of 500-635 nmol/l are indeterminate. Cortisol levels of less

than 500 nmol/l suggest adrenal insufficiency.

Chronic fatigue, dizziness and anxiety along with others are common features with thyroidimbalance (choice A) and most commonly misdiagnosed in people with Addison‟s disease. 

Doxepin (choice E) is a psychotropic agent with tricyclic antidepressant and anxiolytic properties

generally used for patients suffering from depression.

Diabetes is often co-existent with Addison‟s disease. The symptoms typically associated with Addison‟s disease are due absence of adrenal hormones and hence insulin is administered only ifdiabetes is also observed.

Hypersensitivity refers to excessive, undesirable (damaging, discomfort-producing and sometimesfatal) reactions produced by the normal immune system Hypersensitivity reactions require a pre

Page 86: Clinical Immunology and Allergy

7/25/2019 Clinical Immunology and Allergy

http://slidepdf.com/reader/full/clinical-immunology-and-allergy 86/156

fatal) reactions produced by the normal immune system. Hypersensitivity reactions require a pre-sensitized (immune) state of the host. Hypersensitivity reactions can be divided into four types: typeI, type II, type III and type IV, based on the mechanisms involved and time taken for the reaction.Frequently, a particular clinical condition (disease) may involve more than one type of reaction.

Which type of hypersensitivity reaction does the Mantoux test measure?1)  Type I 2)  Type II 3)  Type III 4)  Type IV 

5)  Mixed type of reaction (III and IV)  

The correct answer is choice D

Page 87: Clinical Immunology and Allergy

7/25/2019 Clinical Immunology and Allergy

http://slidepdf.com/reader/full/clinical-immunology-and-allergy 87/156

 

The Mantoux test is a skin test used to test for exposure to Mycobacterium tuberculosis. It is also

known as the PPD ( Purified Protein Derivative) test. It is a skin test which, if positive, revealsprevious exposure to the M. tuberculosis  antigen. The test itself is performed by injecting a smallquantity of protein antigen subcutaneously and measuring the reaction ~48 hours later. Falsepositives may occur if the patient has been vaccinated with the BCG vaccine, is being treated with

Page 88: Clinical Immunology and Allergy

7/25/2019 Clinical Immunology and Allergy

http://slidepdf.com/reader/full/clinical-immunology-and-allergy 88/156

  Patients at low risk: > or = 15mm

Page 89: Clinical Immunology and Allergy

7/25/2019 Clinical Immunology and Allergy

http://slidepdf.com/reader/full/clinical-immunology-and-allergy 89/156

Table 1: Review of Hypersensitivity Reactions 

 A 30 year old man presented to your outpatient clinic complaining of weight loss, night sweats, low

grade fever and general malaise. Examination revealed bilateral cervical and axillary

lympadenopathy but hepatomegaly and splenomegaly were however, absent. White cell count wasnormal, hemoglobin was low and erythrocyte sedimentation rate (ESR) was 75mm/h (normal value:

male < 10 mm/h). Histological examination of cervical lymph nodes revealed the presence of large

binucleate cells. Bone marrow cells were normal and involvement of other lymph nodes was not

seen

Page 90: Clinical Immunology and Allergy

7/25/2019 Clinical Immunology and Allergy

http://slidepdf.com/reader/full/clinical-immunology-and-allergy 90/156

seen.

What is the likely diagnosis for this patient?

1)  Stage I Hodgkin‟s lymphoma 2)  Stage II Hodgkin‟s lymphoma „A‟ 3)  Stage II Hodgkin‟s lymphoma „B‟ 4)  Stage I Non-Hodgkin‟s lymphoma 5)  Burkitt‟s lymphoma 

 

The correct answer is choice C.

The large binucleate cells called Reed-Sternberg cells are characteristic feature of the Hodgkin‟sdisease or Hodgkin‟s lymphoma. This patient is diagnosed as having a stage 2 Hodgkin's disease,because, although only lymphoid tissue above the diaphragm is involved (since spleen and liver arenot enlarged), his ESR is above 40mm/h. Also since lymphoma is no longer asymptomatic, but thepatient experiences some symptoms, the suffix 'B' is added to the stage which suggests a poorerprognosis associated with systemic symptoms.

 

The Ann Arbor staging scheme is a common method for classification of Hodgkin‟s lymphoma. According to this classification, stage I (choice A) is involvement of a single lymph node region mostcommonly the cervical region or a single extralymphatic site. Stage II involves two or more lymph

Page 91: Clinical Immunology and Allergy

7/25/2019 Clinical Immunology and Allergy

http://slidepdf.com/reader/full/clinical-immunology-and-allergy 91/156

commonly the cervical region or a single extralymphatic site. Stage II involves two or more lymphnode regions on the same side of the diaphragm or of one lymph node region and a contiguousextralymphatic site. The stage III Hodgkin‟s lymphoma involves lymph node r egions on both sides

of the diaphragm, which may include the spleen and/or limited contiguous extralymphatic organ orsite. In the stage IV is disseminated involvement of one or more extralymphatic organs. Theabsence of systemic symptoms is signified by adding 'A' to the stage; the presence of systemicsymptoms is signified by adding 'B' to the stage. For localized extranodal extension from mass ofnodes that does not advance the stage, subscript 'E' is added.

Thus, a stage II Hodgkin‟s lymphoma A (choice B) would involve two or more lymph organs without

development of any symptoms.

Most lymphomas that are not Hodgkin‟s type are classified as of non-Hodgkin‟s type (absence ofReed-Sternberg cells) and include about 10 different types of disease (choice D).

The translocation of cmyc leads to constitutive expression of c-Myc and an aggressive, highly

proliferative B-cell lymphoma called Burkitt‟s lymphoma (choice E). Burkitt‟s lymphoma can bediagnosed by t(8;14) c-myc translocation.

 Figure 1: Summary of Ann Arbor staging for Lymphoma

Page 92: Clinical Immunology and Allergy

7/25/2019 Clinical Immunology and Allergy

http://slidepdf.com/reader/full/clinical-immunology-and-allergy 92/156

Page 93: Clinical Immunology and Allergy

7/25/2019 Clinical Immunology and Allergy

http://slidepdf.com/reader/full/clinical-immunology-and-allergy 93/156

 

Page 94: Clinical Immunology and Allergy

7/25/2019 Clinical Immunology and Allergy

http://slidepdf.com/reader/full/clinical-immunology-and-allergy 94/156

The correct answer is choice E.

Rho (D) antigens are not expressed on endothelial tissue and therefore play no apparent role in

Page 95: Clinical Immunology and Allergy

7/25/2019 Clinical Immunology and Allergy

http://slidepdf.com/reader/full/clinical-immunology-and-allergy 95/156

graft rejection or survival. An organ from a donor with ABO type B positive can be safelytransplanted into a recipient with ABO type B negative.

Histocompatibility testing

"Histo" means "tissue." Histocompatibility is defined as a measure of how well 2 tissues "get alongwith one another" when they find themselves in a confined space. Histocompatibility testing is usedto minimize graft foreignness and reduce donor-specific immune responses to the transplantedorgan. The ABO and HLA systems have been identified as the major transplantation antigens inman.

Major histocompatibilitycomplex proteins, or MHC proteins serve as carriers of antigenic material onthe surface of APCs. In humans, the MHC gene loci is divided into three regions: Class I, II, and III.In humans, MHC genes are referred to as human leukocyte antigen (HLA) genes. There are threedifferent important class I MHC genes in humans and they are called HLA-A, B and C. HLA-D

genes belong to MHC class II which include HLA-DP, DQ, and DR. When 2 people share the sameHLAs, they are said to "match." In other words, their tissues are immunologically compatible witheach other. Typing for these MHC class I antigens (choice C) as well as for MHC class II antigens(choice A) is important in determining compatibility for organ transplantation. Many studies haveshown a stepwise decrease in graft survival of cadaver kidneys with increasing numbers of HLAmismatch. The superior results with HLA-A, HLA-B, and HLA-DR mismatches have led to a system

f f

Page 96: Clinical Immunology and Allergy

7/25/2019 Clinical Immunology and Allergy

http://slidepdf.com/reader/full/clinical-immunology-and-allergy 96/156

of mandatory sharing of such donor kidneys.

 At least 15 different minor red cell antigen systems have been identified in humans. The mostimportant of these appears to be the Lewis system. (choice B). Transplant recipients who are highlysensitized to minor red cell antigens as a result of numerous blood transfusions, for example, mayexperience antibody-mediated rejection responses (hyperacute or chronic rejection). For thisreason, the potential recipient's blood is screened for the presence of antibodies to the known

minor red cell antigens before transplantation.

 ABO typing (choice D) detects ABO compatibility. Basic ABO compatibility depends on thepresence or absence of antigens on donor RBCs and the presence or absence of specificantibodies to these antigens in the recipient's serum. Anti-ABO antibodies are of the IgMclassification and cause agglutination, complement fixation, and hemolysis. If an ABO-incompatible

graft is transplanted, hyperacute rejection will occur. In kidney transplantation, preformed circulatingcytotoxic antibodies in the recipient react with ABO isoagglutinins produced by the graft, and thegraft quickly turns dark and soft as a result of diffuse thrombosis of the microvasculature.

  A 76-year-old man presents in your office with a 4-month history of oculodynia, blurred vision, and

weight loss of 4.5 kg. He had no fever, chills, or diaphoresis. A complete blood count revealed

leukocytosis with 98% lymphocytes. His past medical history and family history were unremarkable.

Physical examination revealed a visual field defect peripherally. He had splenomegaly but no

Page 97: Clinical Immunology and Allergy

7/25/2019 Clinical Immunology and Allergy

http://slidepdf.com/reader/full/clinical-immunology-and-allergy 97/156

adenopathy, hepatomegaly, or skin lesions. Additional blood studies showed a positive direct

Coombs, but haptoglobin and bilirubin were normal. A bone marrow biopsy was performed. Flow

cytometry was performed on the peripheral blood and bone marrow aspirate for

immunophenotyping. The lymphocytes were positive for IgM and IgD and were negative for CD3,

CD4 and CD8.

Select the correct answer for the type of cells present.

Page 98: Clinical Immunology and Allergy

7/25/2019 Clinical Immunology and Allergy

http://slidepdf.com/reader/full/clinical-immunology-and-allergy 98/156

 

The correct answer is choice C

Page 99: Clinical Immunology and Allergy

7/25/2019 Clinical Immunology and Allergy

http://slidepdf.com/reader/full/clinical-immunology-and-allergy 99/156

B cell maturation proceeds in antigen-independent and antigen-dependent stages. Antigen-

independent maturation includes development from precursor cells in the marrow through the naiveB cell (a cell that has not been exposed to antigen previously) found in the peripheral lymphoidtissues. Antigen-dependent maturation occurs following the interaction of antigen with naive B cells.Once a B cell encounters its cognate antigen and receives an additional signal from a T helper cell,it can further differentiate into plasma B cells and memory B cells. Plasma cells are terminallydifferentiated B cells responsible for synthesis and secretion of immunoglobulin. The final productsof B cell development are circulating long-lived memory B cells and plasma cells found

predominantly in primary follicles and germinal centers of the lymph nodes and spleen.

Each B cell has a unique receptor protein (referred to as the B cell receptor (BCR)) which is amembrane-bound immunoglobulin on its surface that will bind to one particular antigen.Immunoglobulins (Ig) or antibodies (Ab) are γ-globulins that are produced by plasma cells inresponse to an immunogen. Igs occur in two forms: a soluble form secreted into the blood and

other fluids and a membrane-bound form that is attached to the surface of a B cell. The membrane-bound form of an antibody is part of the BCR, which allows a B cell to detect a specific antigen andtriggers B cell activation. The BCR is composed of surface-bound IgD or IgM antibodies. 

Page 100: Clinical Immunology and Allergy

7/25/2019 Clinical Immunology and Allergy

http://slidepdf.com/reader/full/clinical-immunology-and-allergy 100/156

 B cells have been commonly identified by other surface markers in addition to immunoglobulins.These include the receptor for the Fc portion of immunoglobulins, B cell-specific antigens CD19 andCD20, and surface antigens coded for by the HLA-D genetic region in humans. All mature B cellsbear surface immunoglobulin that is the antigen-specific receptor. The major role of B cells isdifferentiation to antibody-secreting plasma cells. However, B cells may also release cytokines andfunction as antigen-presenting cells

Page 101: Clinical Immunology and Allergy

7/25/2019 Clinical Immunology and Allergy

http://slidepdf.com/reader/full/clinical-immunology-and-allergy 101/156

function as antigen-presenting cells. 

CD4-, CD8- and CD4+, CD8+( choice D) are immature T cells. Mature cytotoxic T-cells are CD4-,CD8+ and MHC class I restricted (choice A) while mature helper T-cells are CD4+, CD8- and MHCclass II restricted (choice B). T-cell receptor (TCR) complex contains an accessory molecule, calledCD3, that is essential of signal transduction. All T cells are positive for CD3. 

Page 102: Clinical Immunology and Allergy

7/25/2019 Clinical Immunology and Allergy

http://slidepdf.com/reader/full/clinical-immunology-and-allergy 102/156

Page 103: Clinical Immunology and Allergy

7/25/2019 Clinical Immunology and Allergy

http://slidepdf.com/reader/full/clinical-immunology-and-allergy 103/156

is no longer recommended before administering MMR vaccine.

The 1994 guidelines (no longer followed) also stated that in allergic children vaccine should begiven in six doses with each dose having increasing amounts of vaccine in it (choice C).

Page 104: Clinical Immunology and Allergy

7/25/2019 Clinical Immunology and Allergy

http://slidepdf.com/reader/full/clinical-immunology-and-allergy 104/156

People with allergies to chickens or feathers have never been shown to be at increased risk forreaction to the vaccine so delaying the vaccine (choice D) or desensitization to egg protein (choiceE), which is not very easy, is not advised.

 A 50-year-old man with end stage renal disease from long-standing diabetes mellitus receives a

cadaveric renal transplant. A month later, he has increasing malaise. His urine output decreases.Laboratory studies show a rising serum urea nitrogen and creatinine. The allograft is biopsied and

seen microscopically to be undergoing destruction by cells which are recognizing graft cells

expressing class I HLA antigens.

Which of the following cells is most likely to mediate acute cellular rejection of solid organ

transplants?

1)  B cells 2)  Eosinophils 

3)  Basophils 4)  T cells 5)  Dendritic cells 

 

The correct answer is choice D

Page 105: Clinical Immunology and Allergy

7/25/2019 Clinical Immunology and Allergy

http://slidepdf.com/reader/full/clinical-immunology-and-allergy 105/156

 Acute cellular rejection is mediated by the T cells. CD4 and CD8 cells recognize the allo- (foreign)antigens on the graft and trigger carefully orchestrated and complex web of changes in thecytokines (e.g., IL2), enzymes (e.g., hydrolysis of inositol phospholipid), minerals (e.g., calcium),and other actors that regulate the cellular immune response. Cytotoxic T cells (CTL) are activatedand release mediators that directly target the foreign cells through a variety of mechanisms. 

Experience with allograft programs and research into the biology of graft rejection has identifiedstrategies for minimizing the immunogenicity of the graft. For example, brain death in the donor hasbeen shown to result in hemodynamic instability and to cause release of cytokines and othermediators in the organ to be transplanted; these changes put the organ into a „proinflammatory‟state. Recent research has shown that treating the donor, after death, with substances such as

catecholemines can interfere with development of this „proinflammatory‟ state in the donor organand can improve graft survival. 

B cells that are the foundation of humoral (antibody) immunity can play a role in rejection buthumoral rejection is differentiated from acute cellular rejection both by the mechanism (B cell vs. Tcell) and also by the clinical consequences. Humoral immunity is responsible for hyperacuterejection that occurs in the first minutes to hours following transplantation. Eosinphils primary role isto combat parasitic and viral infections; they also infiltrate the lungs of asthmatic patients. Basophilsalso play a role in parasitic infections as well as in allergy. Dendritic cells are found in the surfaces

Page 106: Clinical Immunology and Allergy

7/25/2019 Clinical Immunology and Allergy

http://slidepdf.com/reader/full/clinical-immunology-and-allergy 106/156

p y p gyof the body (skin, gut) and act as antigen presenting cells. 

 Acute cellular rejection is the most common form of rejection. It develops generally within the firstmonth after surgery but it may appear at any time, even years after immunologic quiescence.Clinical signs are fever, graft tenderness and pain, oliguria and rapid functional deterioration withproteinuria. With the modern immunosuppressive regimens acute rejection has become lessfrequent and generally less severe so that most of the aforementioned clinical signs may be absent.The rapid increase of serum creatinine level (>25% of the basal value) remains the most commonsign. A T cell response to donor HLA antigens is considered to have a major role in thepathogenesis of this form of rejection. 

 A 27-year-old woman is given intravenous penicillin to treat infective endocarditis. Within minutes of

starting this therapy, she begins to have severe difficulty breathing with respiratory stridor and

tachypnea. She suddenly develops an erythematous skin rash over most of her body.

Page 107: Clinical Immunology and Allergy

7/25/2019 Clinical Immunology and Allergy

http://slidepdf.com/reader/full/clinical-immunology-and-allergy 107/156

 

True IgE-mediated drug allergies are rare. Several studies have shown that only a small fraction ofpatients evaluated for allergy after experiencing an adverse reaction to penicillin had confirmedpenicillin allergy. True IgE-mediated reactions are expected to become more severe with time. 

Page 108: Clinical Immunology and Allergy

7/25/2019 Clinical Immunology and Allergy

http://slidepdf.com/reader/full/clinical-immunology-and-allergy 108/156

Type 2 reactions, cell-mediated responses, are rare and would manifest more slowly. A Type 4reaction would be expected to manifest as contact dermatitis and is most frequently experienced byhealthcare workers who have frequent exposure to penicillin. 

Table 1. Comparison of Different Types of hypersensitivity reactions 

Page 109: Clinical Immunology and Allergy

7/25/2019 Clinical Immunology and Allergy

http://slidepdf.com/reader/full/clinical-immunology-and-allergy 109/156

 

 A 3 year old girl was referred to the hospital by her pediatrician for immunological tests. She

suffered from repeated infections and had had several episodes of otitis media and pneumonia.

Immunological tests revealed the following information.

  IgM 440 mg dl –1 (normal 40 –240 mg dl –1)

  IgG 50 mg dl –1 (normal 639 –1344 mg dl –1)  IgA levels undetectable (normal 70 –312 mg dl –1)

Page 110: Clinical Immunology and Allergy

7/25/2019 Clinical Immunology and Allergy

http://slidepdf.com/reader/full/clinical-immunology-and-allergy 110/156

 

Hyper IgM which results from defects in the AID gene or the gene encoding uracil-DNA glycosylase(UNG) (choice C) and resembles X-linked agammaglobulinemia in that these patients haveincreased susceptibility to pyogenic infections only and not to opportunistic infections such asPneumocystis carinii and Cryptosporidium, which are characteristic of the X-linked hyper IgM

d

Page 111: Clinical Immunology and Allergy

7/25/2019 Clinical Immunology and Allergy

http://slidepdf.com/reader/full/clinical-immunology-and-allergy 111/156

syndrome.

The enzyme activation-induced cytidine deaminase‟ (AID) converts cytidine to uridine. It triggers aDNA repair mechanism that induces isotype switching. B cells that lack functional AID are unable togenerate somatic mutations after activation and therefore cannot undergo affinity maturation. Theonly response these B cells can make to the activating signals is to proliferate. This results in theaccumulation of IgM+ B cells in the lymphoid organs. The B-cell response to T-independentantigens, however is unaffected by this defect, accounting for the production of IgM (choice A),giving rise to an enlarged spleen (splenomegaly) and enlarged lymph nodes (lymphadenopathy)(choice D). AID deficiency causes absence of isotype switching (choice E).

 A 16 year old girl is brought to hospital by her frantic parents after a bee sting. Vital signs are BP,70/40; RR 30 and laboured; HR 140; T 37.5

oC.

Which of the following would not be an appropriate option in her management?

1)  Epinephrine 2)  Diphenhydramine 3)  Methylprednisolone 4)  salbutomol/albuterol 5)  atropine 

f

Page 112: Clinical Immunology and Allergy

7/25/2019 Clinical Immunology and Allergy

http://slidepdf.com/reader/full/clinical-immunology-and-allergy 112/156

6)   All of the above are appropriate measures  

The correct answer is choice E

The hypotension, tachycardia, fever and hyperventilation following a bee sting indicate ananaphylactic (type 1) reaction. Atropine is an anti-cholinergic drug and is contraindicated. Adverseeffects of atropine include tachycardia, temperature effects and arrhythmias. 

 Anaphylaxis is a severe allergic reaction that is rapid in onset and may be fatal. Prominent dermaland systemic signs and symptoms are usually present. The full-blown syndrome may consist

of urticaria (hives) and/or angioedema along with hypotension and bronchospasm. The classic formwas described in 1902 and involves the prior sensitization of an individual to an allergen. Later re-exposure produces symptoms via an immunologic mechanism.

Page 113: Clinical Immunology and Allergy

7/25/2019 Clinical Immunology and Allergy

http://slidepdf.com/reader/full/clinical-immunology-and-allergy 113/156

Page 114: Clinical Immunology and Allergy

7/25/2019 Clinical Immunology and Allergy

http://slidepdf.com/reader/full/clinical-immunology-and-allergy 114/156

Page 115: Clinical Immunology and Allergy

7/25/2019 Clinical Immunology and Allergy

http://slidepdf.com/reader/full/clinical-immunology-and-allergy 115/156

 Figure 1: Anaphylaxis algorithm

Page 116: Clinical Immunology and Allergy

7/25/2019 Clinical Immunology and Allergy

http://slidepdf.com/reader/full/clinical-immunology-and-allergy 116/156

 According to current theory, the most likely pathologic mechanism of insulin-dependent (type I)diabetes mellitus is:

1)  chromosomal translocation 2)  an enzyme defect in the red blood cells 3)  autoimmune destruction of beta cell in the pancreas 4)  high carbohydrate diet over a long period of time 5)  chronic emotional stress 6) autoimmune destruction of alpha cell in the pancreas

Page 117: Clinical Immunology and Allergy

7/25/2019 Clinical Immunology and Allergy

http://slidepdf.com/reader/full/clinical-immunology-and-allergy 117/156

6)  autoimmune destruction of alpha cell in the pancreas 

 

The correct answer is Choice C

Type 1 diabetes is the most common severe chronic autoimmune endocrine disease and accounts

for 90% of childhood-onset diabetes. It also comprises between 5% to 10% of adult diabetes. Inthe US, there are approximately 30,000 new cases diagnosed annually with 1 in 300 children and 1in 100 adults affected. In the US and around the world, the incidence of type 1 diabetes isincreasing at approximately 2% to 5% per year. Furthermore, the median age of diagnosis isdecreasing. Between 10 and 20 million people affected worldwide with significant social andeconomic costs.

Type I diabetes, also known as Insulin Dependent Diabetes Mellitus (IDDM) is an autoimmunedisorder where the insulin producing ß cells of the pancreas are destroyed in a T-cell

Page 118: Clinical Immunology and Allergy

7/25/2019 Clinical Immunology and Allergy

http://slidepdf.com/reader/full/clinical-immunology-and-allergy 118/156

Page 119: Clinical Immunology and Allergy

7/25/2019 Clinical Immunology and Allergy

http://slidepdf.com/reader/full/clinical-immunology-and-allergy 119/156

5)  Langerhans cells are dendritic cells found in the epidermis of the skin  

The correct answer is choice B.

Page 120: Clinical Immunology and Allergy

7/25/2019 Clinical Immunology and Allergy

http://slidepdf.com/reader/full/clinical-immunology-and-allergy 120/156

Only professional antigen-presenting cells (macrophages, B lymphocytes, and dendritic cells) areable to activate a resting helper T-cell when the matching antigen is presented. However,macrophages and B cells can only activate memory T cells whereas dendritic cells can activateboth memory and naive T cells, and are the most potent of all the antigen-presenting cells. Hencechoice B is not a true statement.

Dendritic cells (DCs), are the most potent of all the antigen-presenting cells and represent thepacemakers of the immune response (choice A). They are crucial to the presentation of peptidesand proteins to T and B lymphocytes and are widely recognized as the key antigen presenting cells(APCs). They are critical for the induction of T cell responses resulting in cell-mediated immunity(CMI). The T cell receptors on T lymphocytes recognize fragments of antigens bound to moleculesof the major histocompatibility complex (MHC) on the surfaces of APCs. The peptide bindingproteins are of two types, MHC class I and II, which interact with and stimulate cytotoxic T

lymphocytes (CTLs) and T helper cells (Ths), respectively.

DCs may develop from a myeloid or lymphoid lineage (choice C). The myeloid pathway ofdifferentiation gives rise to DCs that home to peripheral tissues to take up and process exogenous Ags prior to migrating to the secondary lymphoid tissues to present Ags to naïve T cells. Lymphoid(thymic) DCs, on the other hand, promote negative selection in the thymus. They are involved in thepresentation of self-Ag to developing thymocytes and, hence, the subsequent deletion ofautoreactive T cells. Precursors of thymic DCs to migrate to the thymus in an immature form andundergo development exposed only to self-Ag within the thymus.

Page 121: Clinical Immunology and Allergy

7/25/2019 Clinical Immunology and Allergy

http://slidepdf.com/reader/full/clinical-immunology-and-allergy 121/156

Dendritic cells are a major source of many cytokines, namely, interferon-alpha (IFN-a), IL-1, IL-6,IL-7, IL-12, IL-15 etc., all of which are important in the elicitation of a primary immune response.Interleukin-12 production is critical for the promotion of an effective cellular immune response byactivating and differentiating T lymphocyte to the Th 1 pathway (choice D). Its secretion appears tobe inhibited by various tumour-derived substances, including nitric oxide (NO), prostaglandin E2

(PGE2).

Langerhans cells are dendritic cells in the epidermis, containing large granules called Birbeckgranules (choice E). They are normally present in lymph nodes and other organs, including thestratum spinosum layer of the epidermis. They can be found elsewhere, particularly in association

with the condition langerhans cells histiocytosis (a rare disease involving clonal proliferation oflangerhans cells related to other forms of abnormal proliferation of WBCs, such as leukemias andlymphomas).

Page 122: Clinical Immunology and Allergy

7/25/2019 Clinical Immunology and Allergy

http://slidepdf.com/reader/full/clinical-immunology-and-allergy 122/156

diseases caused by many microorganisms 

 A type III hypersensitivity reaction is mediated by soluble immune complexes. They are mostly ofthe IgG class, although IgM may also be involved. The antigen may be exogenous (chronicbacterial, viral or parasitic infections), or endogenous (non-organ specific autoimmunity). Theantigen is soluble and not attached to the organ involved. Primary components are soluble immune

Page 123: Clinical Immunology and Allergy

7/25/2019 Clinical Immunology and Allergy

http://slidepdf.com/reader/full/clinical-immunology-and-allergy 123/156

complexes and complement (C3a, 4a and 5a). The damage is caused by platelets and neutrophilsand lesions contain primarily neutrophils and deposits of immune complexes and complement.Macrophages infiltrating in later stages may be involved in the healing process. 

The affinity of antibody and size of immune complexes are important in production of disease anddetermining the tissue involved. Diagnosis involves examination of tissue biopsies for deposits of

immunoglobulin and complement by immunofluorescence microscopy. The immunofluorescentstaining in type III hypersensitivity is granular (as opposed to linear in type II). The presence ofimmune complexes in serum and depletion in the level of complement are also diagnostic.Polyethylene glycol-mediated turbidity (nephelometry) binding of C1q and Raji cell test are utilizedto detect immune complexes. Treatment includes anti-inflammatory agents. 

Type I hypersensitivity (choice A) is characterized by an immediate anaphylactic response. 

Type II hypersensitivity (choice B), also known as cytotoxic hypersensitivity, is not characterized byimmune complex deposition. 

Type IV hypersensitivity (choice D), also known as cell mediated or delayed type hypersensitivity, isnot typical for cell mediated immunity. 

Page 124: Clinical Immunology and Allergy

7/25/2019 Clinical Immunology and Allergy

http://slidepdf.com/reader/full/clinical-immunology-and-allergy 124/156

Graft versus host disease (choice E) is a rare disorder that occurs when the immune cells fromdonated bone marrow stem cells or a blood transfusion attack the recipient. It does notcharacterized by immune complex deposition. 

Page 125: Clinical Immunology and Allergy

7/25/2019 Clinical Immunology and Allergy

http://slidepdf.com/reader/full/clinical-immunology-and-allergy 125/156

  A 4 year old boy was sent to your clinic for detailed immunological examination. The boy was a

normal term delivery with no complications with proper prenatal care. At one month of age, a

staphylococcal abcess on his chest wall had to be treated surgically along with a course of penicillin

antibiotic. In total, he has developed staphylococcal abscess 5 times before the age of one year.

His development status was below the third percentile. Examination showed a tendency to be

pyrexial, with bilateral axillary and inguinal lymphadenopathy and marked hepatosplenomegaly.

Family history revealed the following facts:

The child had 2 elder sisters, who were healthy; two male siblings had died of infections at 8

months and 2 years. Immunological investigation of showed mild anemia (Hb 105g/l; normal (135-

175 g/L)), IgG and IgA immunoglobulin subclasses showed gross polyclonal elevation. Neutrophils

showed impaired killing of staphylococci. The polymorphs failed to consume oxygen or to produce

hydrogen peroxide during phagocytosis. Thus, a diagnosis of chronic granulomatous disease

(CGD) was made.

Page 126: Clinical Immunology and Allergy

7/25/2019 Clinical Immunology and Allergy

http://slidepdf.com/reader/full/clinical-immunology-and-allergy 126/156

Chronic granulomatous disease is characterized by which of the following traits?

1)  X-linked agammaglobulinemia 2)  Defective T-cells and platelets 3)   Absence of class II MHC molecules 4)  Defect in the oxidative pathway 5)  Hypersensitivity reaction 

 

The correct answer is choice D.

CGD is a genetic disease with two distinct forms; an X-linked form that is seen in about 70% ofpatients and an autosomal recessive form found in the remainder. This disease is caused due to a

defect in the oxidative pathway by which phagocytes generate hydrogen peroxide and the resultingreactive products, for e.g. hypochlorous acid, that kill phagocytosed bacteria. CGD patientsundergo excessive inflammatory reactions that may result in gingivitis, swollen lymph nodes, and

nonmalignant granulomas (lumpy subcutaneous cell masses); they are also susceptible to bacterial

and fungal infection. Several related defects may also lead to CGD; these include a missing ordefective cytochrome (cyt b558) that functions in an oxidative pathway and defects in proteins(phagocyte oxidases, or phox) that stabilize the cytochrome. Also, the general defect in the killerfunction of phagocytes, there is also a decrease in the ability of mononuclear cells to serve as APCs.

Page 127: Clinical Immunology and Allergy

7/25/2019 Clinical Immunology and Allergy

http://slidepdf.com/reader/full/clinical-immunology-and-allergy 127/156

Both processing and presentation of antigen are impaired in this condition. Increased amounts ofantigen are required to trigger T-cell help when mononuclear cells from CGD patients are used as APCs. X-linked agammaglobulinemia (XLA) (choice A) or Bruton‟s hypogammaglobulinemia is a B-cell defect characterized by extremely low IgG levels (not seen in this patient) and by the absenceof other immunoglobulin classes. This is because individuals with XLA have no peripheral B cells.

Defective t-cells and platelets are seen in the Wiskott-Aldrich syndrome (choice B) and involves aspecific defect in the cytoskeletal glycoprotein present in lymphoid cells called sialophorin(CD43).This protein is required for assembly of actin filaments required for the formation ofmicrovesicles. Also, low IgM and platelets are observed.

 Absence of class II MHC molecules (choice C) causes the bare lymphocyte defect. It is also a typeof immunodeficiency. Without these molecules, the patient‟s lymphocytes cannot participate incellular interactions with T helper cells.

 

Hypersensitivity (choice E) reactions are caused by exaggerated immune responses such asoverproduction of cytokines etc. Hypersensitivity reactions do not involve defects in the oxidativepathway.

The basic immunoglobulin molecule has a four-chain structure, comprising of two identical heavy

(H) chains and two identical light (L) chains, linked together by inter-chain disulfide bonds. The

Page 128: Clinical Immunology and Allergy

7/25/2019 Clinical Immunology and Allergy

http://slidepdf.com/reader/full/clinical-immunology-and-allergy 128/156

aminoterminal portions of the heavy and light chains, characterized by a highly variable amino-acid

composition, are referred to a VH and VL, respectively. The constant parts of the light chain are

designated as CL, while the constant parts of the heavy chains are further divided into three distinct

subunits: CH1, CH2 and CH3. The light chains are bound to the heavy chains by many noncovalent

interactions and by disulfide linkages.

The function of the Variable region of the light chain (CL) includes which of the following:

1)  Variation between different immunoglobulin isotypes 2)  Formation of the complementarity determining region 3)  Felicitating antigen binding by attaching to mast cells 4)  Holding the heavy and light chains together  5)   Allowing cross-linking of antigens 

 

The correct choice is choice B.

 

The first ~110 amino acids of the amino-terminal region of a light or heavy chain vary greatlyamong antibodies of different specificity. These segments of highly variable sequence are calledVariable (V) regions. All the differences in specificity exhibited by different antibodies can be tracedto differences in the amino acid sequences of V regions. Areas of the V regions calledcomplementarity determining regions (CDRs), and it is these CDRs, on both light and heavy chains,that constitute the antigen binding site of the antibody molecule.

Page 129: Clinical Immunology and Allergy

7/25/2019 Clinical Immunology and Allergy

http://slidepdf.com/reader/full/clinical-immunology-and-allergy 129/156

By contrast, within the same antibody class, far fewer differences are seen when one comparessequences throughout the rest of the molecule. It is important to note here that the CDR is presenton both the heavy and light chains. The variable regions of the heavy and light chains pair in eacharm of the Y of an immunoglobulin molecule to generate two identical antigen-binding sites, whichlie at the tips of the arms of the Y. It is due to this variability that a large repertoire of antibodies,

capable of binding to different antigens is produced.

The constant region of the heavy chain (CH) is responsible for formation of the different isotypes ofan immunoglobulin molecule (choice A).

The constant regions (CH and CL) interact to hold the molecule together and are involved inseveral biological activities, the so-called effector functions such as complement binding,

Page 130: Clinical Immunology and Allergy

7/25/2019 Clinical Immunology and Allergy

http://slidepdf.com/reader/full/clinical-immunology-and-allergy 130/156

For further diagnosis, the following lab tests were carried out:

  Serum vitamin B12: 42ng/l (normal 170-900)

  Folate: 10.3 ng/ml (normal 7-36 nmol/l)

  Total bilirubin: 2.5 mg/dl(normal 0.2-1.9 mg/dL)

   Alanine aminotransferase: 35 U/mL (normal 8-35 U/l)

   Aspartate aminotransferase: 30 U/mL (normal 8-36 U/l).

Page 131: Clinical Immunology and Allergy

7/25/2019 Clinical Immunology and Allergy

http://slidepdf.com/reader/full/clinical-immunology-and-allergy 131/156

What further testing should be done to obtain a definitive diagnosis?

1)   Anti-intrinsic factor antibody screening 2)  Bone marrow examination 3)  Hepatitis A screening 4)  Electrocardiogram 5)  Schilling test 

 

The correct answer is choice A.

Preliminary tests show that there is a vitamin B12 deficiency. The patient was admitted with signs of

a moderate hemolytic anemia. His CBC shows that he has a macrocytic anemia. Leukocyte andplatelet counts are on the lower end of the normal range, indicating a developing pancytopenia. Hisincreased bilirubin values support a diagnosis of hemolysis. Based on his serum vitamin B12 and

folate results, he can definitively be diagnosed as having a megaloblastic anemia due to a vitamin

B12 deficiency. There is no signs or investigations suggestive of liver disease. Since the liverenzymes are normal, liver disease is ruled out as a source of jaundice or the macrocytic anemia.Thus, further testing should be screening for antibodies against intrinsic factor. A common cause ofvitamin B12 deficiency in a patient with no previous gastrointestinal history is pernicious anemia.

Loss of gastric parietal cells due to autoimmunity leads to a lack of intrinsic factor. This leads to

Page 132: Clinical Immunology and Allergy

7/25/2019 Clinical Immunology and Allergy

http://slidepdf.com/reader/full/clinical-immunology-and-allergy 132/156

g p yinability to absorb vitamin B12 causing pernicious anemia. Positive testing for antibodies againstintrinsic factor will lead to a diagnosis of pernicious anemia. If Intrinsic factor blocking antibodies arenot found in the serum, the vitamin B12 deficiency is due to some other malabsorption disorder.

 A bone marrow examination (choice B) would reveal megaloblastic anemia, caused due to B12

deficiency, but would still not diagnose the cause.

Hepatitis A (choice C) screening is not recommended here since liver functions are all normal (ALT, AST levels).

The patient shows signs of congestive heart failure due to anemia. An electrocardiogram (ECG)(choice D) would be recommended if serum B12 levels were normal.

 

 A schilling test (choice E) involves 4 stages testing with radioactive cobalamin with/without intrinsicfactor and it is important to note that it was once widely used for diagnosis of pernicious anemia butno longer popular because of availability of easier tests such as antibody screening for intrinsicfactor and gastric parietal cells. Hence, anti-intrinsic factor antibody screening is the preferred testin this case. 

Page 133: Clinical Immunology and Allergy

7/25/2019 Clinical Immunology and Allergy

http://slidepdf.com/reader/full/clinical-immunology-and-allergy 133/156

John, a 2 month old child, was referred to your specialty clinic for a work-up for his failure to thrive

and his recent development of seizures. A chest x-ray and echocardiogram revealed an enlarged

heart and truncus arteriosus with a large ventricular septal defect. DiGeorge syndrome was

suspected. Fluorescent in situ hybridization of the buccal mucosa found a 22q11 deletion,

confirming the diagnosis. He showed the characteristic facial abnormalities, Hypo-parathyroidism

and congenital heart disease associated with the disorder.

Immunity against which of the following factors is usually not absent in the patients with such a

disorder?

1)  Viral infections 2)  Intracellular bacteria 

3)  Extracellular bacteria 4)  Intracellular fungi 5)  Vaccines 

 

The correct answer is choice C.

Children with DiGeorge syndrome, or congenital thymic aplasia, are born without a thymus. Theimmune defect includes a profound depression of T-cell numbers and absence of T-cell responses.

Page 134: Clinical Immunology and Allergy

7/25/2019 Clinical Immunology and Allergy

http://slidepdf.com/reader/full/clinical-immunology-and-allergy 134/156

Since patients lack the T-cell component of the cell-mediated immune system, they are generallyable to cope with infections of extracellular bacteria, that can be mediated through phagocytosisand complement mediated lysis but they cannot effectively eliminate intracellular pathogens.

Their lack of functional cell-mediated immunity results in repeated infections with viruses (choice A).

 Also, intracellular bacteria, and fungi (choice B and D) can cause serious complications, since anypathogen for which the normal immune response requires clearance through antigen processingand presentation is needed can evade the immune response.

The severity of the cell-mediated immunodeficiency in these children is such that even theattenuated virus present in a vaccine (choice E), capable of only limited growth in normalindividuals, can produce life-threatening infections.

The most common symptoms of children with DiGeorge syndrome include the following: 

  Congenital heart disease (40% of individuals), particularly conotruncal malformations(tetralogy of Fallot, interrupted aortic arch, ventricular septal defect, and persistent truncusarteriosus)

Page 135: Clinical Immunology and Allergy

7/25/2019 Clinical Immunology and Allergy

http://slidepdf.com/reader/full/clinical-immunology-and-allergy 135/156

  palatal abnormalities (50%), particularly velopharyngeal incompetence (VPI), submucosalcleft palate, and cleft palate; characteristic facial features (present in the majority ofCaucasian individuals) including hypertelorism.

  learning difficulties (90%) but broad range

  hypocalcemia (50%)(due to hypoparathyroidism)

  significant feeding problems (30%)

  renal anomalies (37%)

( ) ( f )

Page 136: Clinical Immunology and Allergy

7/25/2019 Clinical Immunology and Allergy

http://slidepdf.com/reader/full/clinical-immunology-and-allergy 136/156

  hearing loss (both conductive and sensorineural) (Hearing loss with craniofacial syndromes)

  laryngotracheoesophageal anomalies

  growth hormone deficiency

  autoimmune disorders

  seizures (without hypocalcemia)

 

  skeletal abnormalities

Hypersensitivity refers to undesirable (damaging, discomfort producing and sometimes fatal)

reactions produced by the normal immune system Hypersensitivity reactions require a pre

Page 137: Clinical Immunology and Allergy

7/25/2019 Clinical Immunology and Allergy

http://slidepdf.com/reader/full/clinical-immunology-and-allergy 137/156

reactions produced by the normal immune system. Hypersensitivity reactions require a pre-sensitized (immune) state of the host. Hypersensitivity reactions can be divided into four types: type

I, type II, type III and type IV, based on the mechanisms involved and time taken for the reaction.

Frequently, a particular clinical condition (disease) may involve more than one type of reaction.

Which of the following is an example of type 4 hypersensitivity reaction:

1)  Coagulase test 2)  Mantoux test 3)  Schick test 4)  Eleks test 5)   A and B 6)   A and B and C 7)   All of the above are correct (A, B, C and D) 

 

The correct answer is choice B

Classically, immune reactions have been organized by the Gel and Coombs classification. Usingthis system: 

Page 138: Clinical Immunology and Allergy

7/25/2019 Clinical Immunology and Allergy

http://slidepdf.com/reader/full/clinical-immunology-and-allergy 138/156

   A type 1 reaction or immediate hypersensitivity, is mediated by IgE and histamine

   A type 2 reaction is antibody mediated

   A type 3 reaction is mediated by immune complexes

   A type 4 hypersensitivity reaction or delayed type hypersensitivity is mediated by sensitizedT cells.

 

The tuberculin test, also known as the Mantoux test, is mediated by delayed type hypersensitivity. Asmall amount of immunogenic protein from the mycobacterium is injected under the skin. If theindividual has previously been exposed to TB and has developed immunological memory to thebacterium, a T cell-mediated response ensues. 

Table 1: Delayed hypersensitivity reactions review

Page 139: Clinical Immunology and Allergy

7/25/2019 Clinical Immunology and Allergy

http://slidepdf.com/reader/full/clinical-immunology-and-allergy 139/156

Table 1: Delayed hypersensitivity reactions review 

Table 2. Comparison of Different Types of hypersensitivity

Page 140: Clinical Immunology and Allergy

7/25/2019 Clinical Immunology and Allergy

http://slidepdf.com/reader/full/clinical-immunology-and-allergy 140/156

 

In the coagulase test, coagulase found on the cell walls of certain types of Staph organisms bindsto fibrinogen, causing it to precipitate or to agglutinate the organisms.

The Schick test is no longer used. It was devised to differentiate people who were sensitive to

diptheria toxin from those who had sufficient immunity to protect them from adverse effects. Asmall amount of diptheria toxin would be injected intracutaneously and evaluated after 48 hours. An unprotected individual would have developed an inflammatory reaction by that point; an

Page 141: Clinical Immunology and Allergy

7/25/2019 Clinical Immunology and Allergy

http://slidepdf.com/reader/full/clinical-immunology-and-allergy 141/156

being acute renal failure. 

Type II- Antibody mediated hypersensitivity. Type II hypersensitivity ischaracterized by opsonizationand phagocytosis. Prototypic disorders include auto-immune hemolytic anemias, GoodpastureSyndrome, Graves' Disease, Myesthenia gravis, Type II diabetes, pemphigus and acuterheumatoid arthritis. It is Th1 mediated.

Page 142: Clinical Immunology and Allergy

7/25/2019 Clinical Immunology and Allergy

http://slidepdf.com/reader/full/clinical-immunology-and-allergy 142/156

The other choices are correctly matched. (Choices A,C,D and E). 

Serum sickness occurs with, for example the use of a xenogeneic antivenom for the treatment of a

snake bite (Choice C). A Mantoux test (Choice D) is used to test for reactivity to Mycobacteriumtuberculosis and is considered positive if the indurated zone is >15mm. 

Page 143: Clinical Immunology and Allergy

7/25/2019 Clinical Immunology and Allergy

http://slidepdf.com/reader/full/clinical-immunology-and-allergy 143/156

 A 50-year-old man with end stage renal disease from long-standing diabetes mellitus receives a

cadaveric renal transplant. A month later, he experiences increasing malaise. His urine output

decreases. Laboratory studies show a rising serum urea nitrogen and creatinine. The allograft is

biopsied and seen microscopically to be undergoing destruction by cells which are recognizing graft

cells expressing class I HLA antigens.

Which of the following cells is most likely to mediate acute cellular rejection of solid organ

transplants?

Page 144: Clinical Immunology and Allergy

7/25/2019 Clinical Immunology and Allergy

http://slidepdf.com/reader/full/clinical-immunology-and-allergy 144/156

transplants?

1)  CD4 lymphocytes 2)  NK cells 3)  CD8 lymphocytes 4)  Macrophages 5)  Neutrophils 

 

The correct answer is choice C

Transplant rejection occurs when a transplanted organ or tissue is not accepted by the body of the

transplant recipient. There are three types of transplant rejections that have different time framesand are mediated by different mechanisms: hyperacute, acute, and chronic rejection. 

 

Hyperacute rejection is a complement-mediated response in recipients with pre-existing antibodiesto the donor (for example, ABO blood type antibodies). Hyperacute rejection occurs within minutesand the transplant must be immediately removed to prevent a severe systemic inflammatoryresponse. Rapid agglutination of the blood occurs. This is a particular risk in kidney transplants so aprospective cytotoxic crossmatch is performed prior to kidney transplantation to ensure thatantibodies to the donor are not present. Hyperacute rejection is analogous to a blood transfusionreaction as it is a humoral-mediated immune response. 

Page 145: Clinical Immunology and Allergy

7/25/2019 Clinical Immunology and Allergy

http://slidepdf.com/reader/full/clinical-immunology-and-allergy 145/156

 Acute rejection usually begins one week after transplantation (as opposed to hyperacute rejection,which is immediate). The risk of acute rejection is highest in the first three months aftertransplantation. However, acute rejection can also occur months to years after transplantation. Asingle episode of acute rejection is not a cause for concern if recognized and treated promptly, and

rarely leads to organ failure. It is caused by mismatched HLA, which are present on all cells of thebody. There are a large number of different alleles of each HLA, so a perfect match between allHLA in the donor tissue and the recipient's body is extremely rare. 

Tissues such as the kidney or the liver which are highly vascularized, are often the earliest victimsof acute rejection. In fact, episodes of acute rejection occur in around 10-30% of all kidney

transplants, and 50 to 60% of liver transplants. Damage to the endothelial lining of blood vessels isan early predictor of irreversible acute transplant rejection. CD8+ T-cells are involved in therejection mechanism. The T-cells cause cells in the transplanted tissue to lyse, or produce

cytokines that cause necrosis of the transplanted tissue. 

Chronic rejection occurs many years after transplantion and is due to a poorly understood chronicinflammatory and immune response against the transplanted tissue. Typically, there is fibrosis ofthe internal blood vessels of the transplanted tissue. 

Page 146: Clinical Immunology and Allergy

7/25/2019 Clinical Immunology and Allergy

http://slidepdf.com/reader/full/clinical-immunology-and-allergy 146/156

CD4 lymphocytes (choice A) are involved in humoral-mediated immune response. They areinvolved in chronic rejection of organs. 

NK cells (choice B) are a type of cytotoxic lymphocyte that constitute a major component of the

innate immune system. NK cells play a major role in the rejection of tumors and cells infected byviruses. They kill cells by releasing small cytoplasmic granules of proteins called perforin andgranzyme that cause the target cell to die by apoptosis. They are not involved in acute tissuerejection mechanisms. 

Macrophages (choice D) are phagocytes, acting in both non-specific defense (innate immunity) as

well as to help initiate specific defense mechanisms (adaptive immunity). Their role is tophagocytose (engulf and then digest) cellular debris and pathogens either as stationary or asmobile cells, and to stimulate lymphocytes and other immune cells to respond to the pathogen.

They are involved in chronic rejection of organs. 

Neutrophils (choice E) are the most abundant type of white blood cells. They are active during theacute phase of inflammation, particularly as a result of bacterial infection and some cancers.Neutrophils are one of the first-responders of inflammatory cells to migrate toward the site ofinflammation, firstly through the blood vessels, then through interstitial tissue, following chemicalsignals (such as Interleukin-8 (IL-8) and C5a) in a process called chemotaxis. They are the

predominant cells in pus, accounting for its whitish/yellowish appearance.They are not involved in

Page 147: Clinical Immunology and Allergy

7/25/2019 Clinical Immunology and Allergy

http://slidepdf.com/reader/full/clinical-immunology-and-allergy 147/156

p p g y pp yacute tissue rejection mechanisms. 

The second pregnancy for a 23-year-old woman appears uncomplicated until ultrasound performed

at 19 weeks shows hydrops fetalis. The fetal organ development is consistent for 19 weeks, and no

congenital anomalies are noted. Her first pregnancy was uncomplicated and resulted in the birth of

a normal girl at term. The current pregnancy ends with birth of a baby boy at 32 weeks gestation.

On examination the baby has marked icterus. Laboratory studies show a hemoglobin of 7.5 g/dL,

and an exchange transfusion is performed.

Which of the following immunologic mechanisms best explains this infant's findings?

1)  IgM deficiency 2)  Complement mediated cell destruction 3)  Immune complex formation 

4)  Type 1 hypersensitivity 

5)   Anti-receptor antibody  

The correct answer is choice B.

Until the advent of routine immunization of Rh negative mothers, most immune hydrops was due to

Page 148: Clinical Immunology and Allergy

7/25/2019 Clinical Immunology and Allergy

http://slidepdf.com/reader/full/clinical-immunology-and-allergy 148/156

alloimmunization, resulting in the complement-mediated lysis of the infant‟s red blood cells. This istermed a type II hypersensitivity reaction. Currently, most cases of hydrops are non-immune.However, immune hydrops continues to occur; exposure to foreign RBC on contaminated needlesin illicit drug use has been shown to be responsible for about 20% of current Rh sensitization.These women are particularly at risk. One study showed that the incidence of immune hyrops intheir infants was 80%.

While ultrasound can identify fetal hydrops, it cannot determine the fetus‟ hematocrit. Doppler bloodflow analysis is similarly unable to determine hematocrit. At-risk pregnancies are monitored withrepeated analysis of bilirubin levels in the amniotic fluid. Transfusion is the recommendedtreatment. 

Type II Hypersensitivity Review

 

It is also known as cytotoxic hypersensitivity and may affect a variety of organs and tissues. Theantigens are normally endogenous, although exogenous chemicals (haptens) which can attach tocell membranes can also lead to type II hypersensitivity. Drug-induced hemolytic anemia,granulocytopenia and thrombocytopenia are such examples. The reaction time is minutes to hours.It is primarily mediated by antibodies of IgM or IgG class and complement. Phagocytes and K cellsmay also play a role (ADCC).

Th l i t i tib d l t d t hil Di ti t t i l d d t ti f

Page 149: Clinical Immunology and Allergy

7/25/2019 Clinical Immunology and Allergy

http://slidepdf.com/reader/full/clinical-immunology-and-allergy 149/156

The lesion contains antibody, complement and neutrophils. Diagnostic tests include detection ofcirculating antibody against tissues involved and the presence of antibody and complement in thelesion (biopsy) by immunofluorescence. The staining pattern is normally smooth and linear, such asthat seen in Goodpasture‟s nephritis (renal and lung basement membrane) and pemphigus (skinintercellular protein, desmosome).

Treatment involves anti-inflammatory and immunosuppressive agents. 

Table 1. Comparison of Different Types of hypersensitivity 

Page 150: Clinical Immunology and Allergy

7/25/2019 Clinical Immunology and Allergy

http://slidepdf.com/reader/full/clinical-immunology-and-allergy 150/156

 

 A 41 year old female presented with complaint of excessive sweating, palpitations and unexplained

weight loss of about 5 kg in 2 months. She also complained of increased nervousness and anxiety.

Examination revealed obvious enlargement of thyroid. Her resting pulse rate was high (155/min)

and she was diaphoretic as you examined her.

Laboratory tests revealed raised levels of serum T3 (4nmol/l; normal 1.8-2.9 nmol/l), T4 was also

high at 45nmol/l (normal 9-23 nmol/l). Thyroid secreting hormone was normal at the lower end

(0.4mU/l) (normal range 0.5-5.0 mU/l). Thus, pituitary over activity was ruled out. Circulatingantibodies to thyroid peroxidase were high (1:3000; 200iu/ml). A diagnosis of Graves' disease was

made and the antithyroid drug, carbimazole, was given to control her thyrotoxicosis.

The immunological basis of Graves‟ disease is related to which of the following? 

1)  Expression of class I MHC on thyroid acinar cells 2)  Inappropriate expression of class II MHC on thyroid acinar cells 3) A t i it i t t l h li t

Page 151: Clinical Immunology and Allergy

7/25/2019 Clinical Immunology and Allergy

http://slidepdf.com/reader/full/clinical-immunology-and-allergy 151/156

3)   Autoimmunity against acetylcholine receptors 4)  Inability to absorb iodine 5)  None of the above are true 

 

The correct answer is choice B.

The synthesis of two thyroid hormones, thyroxine (T4) and triiodothyronine (T3) is carefullyregulated by the thyroid-stimulating hormone (TSH) produced by the pituitary gland. A patient withGraves‟ disease produces auto-antibodies that bind the receptor for TSH and mimic the normalaction of TSH, activating adenylate cyclase and resulting in production of the thyroid hormones.Since the binding of auto antibodies is permanent, the thyroid remains overstimulated. The thyroid

acinar cells from patients with Graves‟ disease express class II MHC molecules on theirmembranes. This inappropriate expression of class II MHC molecules, which are normallyexpressed only on antigen presenting cells, may serve to sensitize T Helper cells to peptides

Page 152: Clinical Immunology and Allergy

7/25/2019 Clinical Immunology and Allergy

http://slidepdf.com/reader/full/clinical-immunology-and-allergy 152/156

with flaccid tetraplegia. Two weeks ago, she had fever, weakness, fatigue, and a cough. A

nasopharyngeal swab was performed, and reverse-transcription polymerase chain reaction (RT-PCR) confirmed infection with pandemic flu virus A (H1N1). Ten days later, she suffered muscular

weakness of the lower limb, which then progressed to the upper limb. At admission, she had normal

consciousness, flaccid tetraplegia with complete areflexia, and symmetric paresthesia of the lower

limb. A few hours later, she developed severe dyspnea with impaired coughing, accumulation of

airway secretions, nasal voice, swallowing difficulties, and severe hypoxemia requiring mechanical

ventilation. During the first 2 days, episodes of bradycardia and hypertension were recorded onhemodynamic monitoring Electromyography showed reduction in motor conduction velocity and

Page 153: Clinical Immunology and Allergy

7/25/2019 Clinical Immunology and Allergy

http://slidepdf.com/reader/full/clinical-immunology-and-allergy 153/156

hemodynamic monitoring. Electromyography showed reduction in motor conduction velocity and

prolonged distal latencies, which were more severe in the lower limb.

Subsequently, five consecutive plasma exchanges were performed, and the patient improved

progressively. She was free from mechanical ventilation within 2 weeks, and neither swallowing

problems nor dysautonomia abnormalities were reported. The patient was discharged from theintensive care unit within 45 days.

Which of the following best describes the above scenario?

1)   Amyotrophic lateral sclerosis (ALS) 2)  Guillain-Barré syndrome 3)  Lambert-Eaton myasthenic syndrome (LEMS) 4)  Myasthenia gravis 5)  Inclusion body myositis 

 

The correct answer is choice B.

 As described above the classical symptom of Guillain-Barré syndrome include varying degrees ofweakness or tingling sensations in the legs. In many instances, the weakness and abnormalsensations spread to the arms and upper body. These symptoms can increase in intensity until the

muscles cannot be used at all and the patient is almost totally paralyzed. In these cases, thedisorder is life-threatening and is considered a medical emergency The patient is often put on a

Page 154: Clinical Immunology and Allergy

7/25/2019 Clinical Immunology and Allergy

http://slidepdf.com/reader/full/clinical-immunology-and-allergy 154/156

disorder is life threatening and is considered a medical emergency. The patient is often put on arespirator to assist with breathing.

 All forms of Guillain –Barré syndrome are due to an immune response to foreign antigens (such asinfectious agents) that are mistargeted at host nerve tissues instead. The targets of such immuneattack are usually gangliosides. The most common antecedent infection is the bacteriumCampylobacter jejuni. In 66% of patients, history of respiratory or gastrointestinal infection within 6weeks preceding onset of the disease is found. Some cases are triggered by the influenza virus, orby an immune reaction to the influenza virus. In adults, plasmapheresis is considered the goldstandard for treatment of the most severe cases of Guillain-Barre´ syndrome.

 Amyotrophic lateral sclerosis (ALS) (choice A) is the most common type of adult-onset motorneuron disease. Adult-onset motor neuron diseases are a group of neurologic disorders that are

characterized primarily by progressive degeneration and loss of motor neurons. ALS involves upper

and lower motor neurons resulting in progressive muscle weakness, atrophy, and fasciculations.The disease can be divided into familial and sporadic forms. Familial ALS nearly always istransmitted in an autosomal dominant pattern and a mutation for the gene Cu/Zn superoxidedismutase on chromosome 21or ALS2, has also been found to be associated with the disease.Current research into the mechanisms resulting in sporadic types of ALS has focused onexcitotoxicity. This may occur due to overactivation of glutamate receptors, autoimmunity to calciumion channels, oxidative stressors linked to free radical formation, or even cytoskeletal abnormalitiessuch as intracellular accumulation of neurofilaments.

Page 155: Clinical Immunology and Allergy

7/25/2019 Clinical Immunology and Allergy

http://slidepdf.com/reader/full/clinical-immunology-and-allergy 155/156

Lambert-Eaton myasthenic syndrome (LEMS) (choice C) is a rare condition in which weaknessresults from an abnormality of acetylcholine (ACh) release at the neuromuscular junction. LEMSresults from an autoimmune attack against voltage-gated calcium channels (VGCC) on thepresynaptic motor nerve terminal. Cancer is present when the weakness begins or is later found in40% of patients with LEMS. Symptoms usually begin insidiously. Many patients have symptoms formonths or years before the diagnosis is made. Weakness is the major symptom, with proximalmuscles more affected than distal muscles (especially in the lower limbs).

Myasthenia gravis (MG) (choice D) is an acquired autoimmune disorder characterized clinically byweakness of skeletal muscles and fatigability on exertion. The antibodies in myasthenia gravis are

directed toward the acetylcholine receptor (AChR) at the neuromuscular junction (NMJ) of skeletalmuscles. Myasthenia gravis is characterized by fluctuating weakness increased by exertion.Weakness increases during the day and improves with rest. Presentation and progression may

vary.

Inclusion body myositis (IBM) (choice E) is an inflammatory muscle disease, characterized byslowly progressive weakness and wasting of both distal and proximal muscles, most apparent in themuscles of the arms and legs, involvement of the knee extensors, ankle dorsiflexors, andwrist/finger flexors is characteristic of sporadic IBM, dysphagia is common, as is mild facialweakness.

Page 156: Clinical Immunology and Allergy

7/25/2019 Clinical Immunology and Allergy

http://slidepdf.com/reader/full/clinical-immunology-and-allergy 156/156